Dermatology Amboss Q&A

¡Supera tus tareas y exámenes ahora con Quizwiz!

A 44-year-old African American woman comes to the physician for a routine examination. She is concerned about cancer because her uncle died of metastatic melanoma 1 year ago. She has no history of serious illness and does not take any medication. She has been working in a law firm for the past 20 years and travels to the Caribbean regularly with her husband. Examination of the skin shows no abnormal moles or warts. This woman is at greatest risk of which of the following types of melanoma? Dark-skinned individuals have a significantly lower risk of melanoma than light-skinned individuals but are more commonly affected by a type of melanoma that tends to occur on the palms, soles, nail beds, and mucous membranes.

Acral lentiginous Acral lentiginous melanoma (ALM) is uncommon in individuals of European descent but is the most common type of melanoma in African American and Asian individuals. Unlike other types of melanomas, ALM is not associated with sun exposure. When arising from the nail bed, ALM usually manifests as a dark linear patch that widens with time (Hutchinson nail sign). On the palms, soles, and mucous membranes, ALM appears as an irregularly shaped, brown-black macule. Overall the most common type of melanoma is superficial spreading melanoma, followed by nodular melanoma, and lentigo maligna melanoma.

.A 66-year-old man comes to the physician with a 4-month history of recurrent crusty lesions on the dorsal surface of his hands. When he scrapes off the crusts, they reappear after a few days. The lesions are mildly pruritic and occasionally burn. He works as a landscaper. A photograph of his right hand is shown. Which of the following is the most likely diagnosis? The hand is covered with light brown papules and rough, hyperkeratotic plaques. This condition is the result of years of exposure to sunlight without protection.

Actinic keratosis Actinic keratosis is a precancerous lesion that occurs on sun-exposed skin (e.g., the scalp, face, ears, dorsal aspects of the hands or arms) and predominantly affects light-skinned individuals over 50 years of age. The lesions typically grow back after they are scraped off. As a landscaper, this man has likely spent a lot of time working outdoors and has been frequently exposed to sun. Initially, actinic keratosis present as small, erythematous lesions with a central scale and a rough surface, giving them a sandpaper-like texture on palpation. Later, the lesions may grow and become erythematous and scaly, as seen in this patient. The risk of progression to invasive squamous cell carcinoma depends on the degree of epithelial dysplasia, but it is < 1% per year. Occasionally, hyperkeratosis may turn pre-existing actinic keratosis lesions into cutaneous horns.

A 45-year-old man with HIV comes to the physician because of multiple lesions on his chest and lower extremities. The lesions have progressively increased in size and are not painful or pruritic. Current medications include abacavir, dolutegravir, and lamivudine. A photograph of the lesions is shown. His CD4+ T-lymphocyte count is 450/mm3. A skin biopsy shows multiple spindle-shaped cells and lymphocytic infiltrate. Which of the following is the most appropriate pharmacotherapy? In an HIV-positive patient, multiple painless violaceous skin lesions that progress in size point toward Kaposi sarcoma. Spindle cells and lymphocytic infiltrate on skin biopsy confirm the diagnosis.

Alpha-interferon Alpha-interferon (IFN-α) is a biologic modifier that is effective in the treatment of Kaposi sarcoma. This type of sarcoma can be caused by HHV-8 infection and is an AIDS-defining illness. How IFN-α fights cancer cells is not fully understood; upregulation of the immune response is thought to play a role. IFN-α is also used in the treatment of hepatitis B and C infections.

A 31-year-old woman, gravida 1, para 0, at 10 weeks' gestation comes to the physician because of a rash on her upper arm that appeared 3 days ago. She has also had headaches and muscle aches for 1 day. She went on a camping trip in Maine 10 days ago. Her temperature is 39°C (102.2°F). A photograph of her rash is shown. Which of the following is the most appropriate pharmacotherapy? This patient's rash is consistent with erythema migrans, which is pathognomonic for early localized Lyme disease.

Amoxicillin Oral amoxicillin is a first-line treatment for Lyme disease in pregnant and nursing women, as well as in children under the age of 8 years. Lyme disease, which is caused by Borrelia burgdorferi and transmitted by the Ixodes deer tick, is endemic to the northeastern US (e.g., Maine).

A 3-year-old boy is brought to the physician for follow-up examination 5 days after sustaining a forehead laceration. Examination shows a linear, well-approximated laceration over the right temple. The wound is clean and dry with no exudate. There is a small amount of pink granulation tissue present. Microscopic examination of the wound is most likely to show which of the following? This boy's laceration, which has been allowed to heal by primary intention, is in the proliferative phase of wound healing.

Angiogenesis with type III collagen deposition Microscopic examination of this boy's wound is most likely to show granulation tissue, which is characterized by angiogenesis (driven by secretion of VEGF, EGF, TGF-β, and PDGF) with type III collagen deposition. There are 4 phases of wound healing: the exudative, resorptive, proliferative, and maturation phases. The proliferative phase (3-7 days) is characterized by the presence of fibroblasts, endothelial cells, keratinocytes, macrophages, and myofibroblasts, which cause wound contraction.

A 16-year-old boy is brought to the physician by his mother because of a 4-day history of painful lesions in the mouth. During the past year, he has twice had similar lesions that resolved without treatment after approximately 10 days. He has never had any genital or anal lesions. His mother reports that he has been very stressed over the past month because he is approaching his senior year in high school. He is otherwise healthy and takes no medications. He appears thin. His temperature is 37.6°C (99.7°F). A photograph of the oral cavity is shown. The remainder of the examination shows no abnormalities. Which of the following is the most likely diagnosis? The image shows two ovoid erosions with regular margins and surrounding erythema on the labial mucosa.

Aphthous stomatitis Aphthous stomatitis is a common condition that manifests as painful, oral mucosal ulcers with regular margins, a yellow fibrinous base, and a surrounding erythematous halo. The lesions occur in parts of the oral cavity with nonkeratinized mucosa, such as the buccal and labial mucosae, the floor of the mouth, and the ventral surface of the tongue. The lesions typically disappear without treatment within 10-14 days but can recur during periods of acute stress.

A previously healthy 24-year-old woman comes to the physician because of a 1-day history of painful rash after spending several hours in the sun. Skin examination shows well-demarcated areas of erythema with some scaling on the face, chest, upper back, and arms. The affected areas are hot and sensitive to touch. The oral mucosa appears normal. Which of the following is the most likely underlying mechanism of this patient's skin findings? UVB-radiation causes DNA-damage of epidermal cells.

Apoptosis of keratinocytes in the epidermis This patient's rash following prolonged sun exposure is consistent with dermatitis solaris, or sunburn. Sunburn is caused by ultraviolet (primarily UVB) radiation-induced damage of the epidermis, which results in keratinocyte apoptosis and release of inflammatory mediators within the epidermis and dermis. Lesions are painful and erythematous and appear within hours of sun exposure. Sunburn is typically restricted to sun-exposed skin, as reflected by involvement of this patient's face, chest, upper back, and arms, and sparing of the oral mucosa.

An otherwise healthy 17-year-old girl comes to the physician because of multiple patches on her face, hands, abdomen, and feet that are lighter than the rest of her skin. The patches began to appear 3 years ago and have been gradually increasing in size since. There is no associated itchiness, redness, numbness, or pain. She emigrated from India 2 years ago. An image of the lesions on her face is shown. Which of the following is most likely involved in the pathogenesis of this patient's skin findings? The presence of sharply demarcated, depigmented skin macules on otherwise normal skin in a patient with no further symptoms is highly suggestive of vitiligo.

Autoimmune destruction of melanocytes Vitiligo is a common skin condition that is thought to be caused by autoimmune destruction of melanocytes. The areas of depigmentation commonly involve the face (e.g., perioral and periocular regions), neck, scalp, acral surfaces (e.g., hands), extensor surfaces, and genitalia. Vitiligo is generally a clinical diagnosis. It may be a sign of an autoimmune disorder (e.g., autoimmune thyroiditis, type 1 diabetes mellitus, pernicious anemia), and patients with vitiligo should be tested for thyroid dysfunction and markers of autoimmune disease (e.g., ANA).

A 62-year-old man comes to the physician because of a 2-month history of an itchy rash and a 7-kg (15-lb) weight loss. Physical examination shows multiple erythematous plaques on the arms, legs, and chest. There are palpable lymph nodes in the axillary and inguinal areas. A biopsy of a skin lesion shows aggregates of neoplastic cells within the epidermis. A peripheral blood smear is most likely to show which of the following findings in this patient? Collections of neoplastic cells in the epidermis (Pautrier microabscesses) are seen in mycosis fungoides.

CD4+ cells with cerebriform nuclei Mycosis fungoides, which is a cutaneous T-cell lymphoma, is the most common primary cutaneous lymphoma. It can progress to Sézary syndrome (T-cell leukemia), which is characterized by systemic symptoms (e.g., weight loss, lymphadenopathy) and the presence of CD4+ cells with cerebriform nuclei in peripheral blood smears.

A previously healthy 30-year-old man comes to the physician because of a 2-week history of skin lesions on his elbows. He has no history of serious illness and takes no medications. Physical examination shows skin lesions on bilateral elbows. A photograph of his right elbow is shown. Which of the following is the most appropriate treatment for this patient's skin condition? The pictured lesions are well-demarcated, erythematous, rounded plaques with silvery-white scale on the extensor surfaces of the arm. These findings are typical of mild plaque psoriasis.

Calcipotriene Calcipotriene, a vitamin D analog, is a first-line topical therapy for the treatment of mild psoriasis. Although its exact mechanism of action is unclear, it activates the intracellular vitamin D receptor, a transcription factor that promotes keratinocyte differentiation and inhibits keratinocyte proliferation. Keratinocytes are characteristically hyperproliferative in plaque psoriasis. Vitamin D analogs are effective when used alone but are commonly used in combination therapy with topical corticosteroids.

A 62-year-old man comes to the physician for evaluation of multiple red spots on his trunk. He first noticed these several months ago, and some appear to have increased in size. One day ago, he scratched one of these spots, and it bled for several minutes. Physical examination shows the findings in the photograph. Which of the following is the most likely diagnosis? This middle-aged man presents with bright red papules that bleed with traumatic rupture, which suggests that the lesions are highly vascular.

Cherry angioma Cherry angiomas are benign, vascular proliferations that commonly develop in adult patients. The diagnosis is typically made based on the clinical appearance of multiple, bright red, dome-shaped papules on the trunk and upper extremities that bleed with trauma and usually blanch with pressure. These lesions manifest more frequently with age and do not regress. Histologically, they are characterized by dilated interconnecting capillaries within the papillary dermis.

cholTwo weeks after undergoing an emergency cardiac catheterization for unstable angina pectoris, a 65-year-old man has decreased urinary output. He takes naproxen for osteoarthritis and was started on aspirin, clopidogrel, and metoprolol after the coronary intervention. His temperature is 38.1°C (100.5°F), pulse is 96/min, and blood pressure is 128/88 mm Hg. Examination shows mottled, reticulated purplish discoloration of the feet and ischemic changes on the right big toe. His leukocyte count is 16,500/mm3 with 56% segmented neutrophils, 12% eosinophils, 30% lymphocytes, and 2% monocytes. His serum creatinine concentration is 4.5 mg/dL. A photomicrograph of a kidney biopsy specimen is shown. Which of the following is the most likely cause of this patient's presentation? Contains overlay This condition can also be seen in patients undergoing endovascular abdominal aortic aneurysm repair.

Cholesterol embolization Cholesterol embolization syndrome is a known complication of cardiac catheterization, vascular interventions, and anticoagulant therapy. Cholesterol is released into the circulation, where it can travel to many organs, especially the kidneys (as they receive a large amount of the blood supply). This patient presents with the classic symptoms of blue toe syndrome, livedo reticularis, and acute kidney failure. Eosinophilia and eosinophiluria in the setting of cholesterol embolism are very suggestive findings. The kidney biopsy shows intravascular cholesterol deposits (spindle-shaped vacuoles) with multiple intra-arterial 'cholesterol clefts' and associated giant-cells, which confirms the diagnosis.

A 62-year-old woman with type 2 diabetes mellitus is brought to the emergency department by her husband because of fever, chills, and purulent drainage from a foot ulcer for 2 days. Her hemoglobin A1c was 15.4% 16 weeks ago. Physical examination shows a 2-cm ulcer on the plantar surface of the left foot with foul-smelling, purulent drainage and surrounding erythema. Culture of the abscess fluid grows several bacteria species, including gram-negative, anaerobic, non-spore-forming bacilli that are resistant to bile and aminoglycoside antibiotics. Which of the following is the most likely source of this genus of bacteria? Culture of this patient's abscess fluid grows Bacteroides species.

Colon This patient most likely has a wound infection with Bacteroides fragilis, an anaerobic bacterium that is normally found in the colon. B. fragilis is a common pathogen in infected diabetic ulcers, particularly in the setting of a polymicrobial infection as seen in this patient. The additional infecting bacteria are necessary, as aerobic bacteria are required to create the tissue conditions necessary for an anaerobic infection.

An 84-year-old woman is brought to the physician by her daughter because of hair loss over the past 2 months. The patient has a history of hypertension, major depressive disorder, allergic rhinitis, osteoarthritis, and diverticular disease. She lived with her daughter, son-in-law, and their 3 children until recently, when her daughter got divorced and had to move into an apartment. The patient was moved to an assisted living facility nearby, and her daughter has not been able to visit as often as planned. Her medications include diltiazem, venlafaxine, loratadine, acetaminophen, and fiber supplements. She appears withdrawn. Her vital signs are within normal limits. Physical examination of the head is shown. There is no scarring of the skin. The remainder of the examination shows no abnormalities. Which of the following is the most likely cause of this patient's hair loss? This patient experienced significant stressors when she was moved away from her family to an assisted living facility after her daughter's divorce.

Compulsive hair pulling This patient's patches of hair of varying lengths that occurred following stressful events suggest trichotillomania. Patients with trichotillomania often have an underlying psychiatric disorder, such as major depressive disorder. Trichotillomania is more common in women than men and manifests most commonly in teenagers, although it can present at any age, particularly in the setting of multiple stressors. Hair-pulling can occur at the scalp, as seen in this patient, as well as eyebrows and eyelashes. Treatment includes cognitive behavioral therapy and SSRIs.

A previously healthy 3-year-old boy is brought to the physician by his parents because of fever and a rash for 6 days. His temperature is 38.9°C (102°F). Examination shows right-sided anterior cervical lymphadenopathy, bilateral conjunctival injection, erythema of the tongue and lips, and a maculopapular rash involving the hands, feet, perineum, and trunk. Which of the following is the most common complication of this patient's condition? The patient presents with signs and symptoms of Kawasaki disease, an acute vasculitis that primarily affects young children and is characterized by a high fever, desquamative rash, conjunctivitis, mucositis, cervical lymphadenopathy, and erythema and edema of the distal extremities.

Coronary artery aneurysm Coronary artery aneurysm is the most common complication of Kawasaki disease. Coronary artery aneurysms occur due to the acute necrotizing vasculitic involvement of the coronary arteries and can lead to myocarditis, pericarditis, arrhythmias, and myocardial infarction, the main cause of death in Kawasaki disease. Other cardiac sequelae such as heart failure due to decreased myocardial contractility, valvular regurgitation, and pericardial effusion can also occur.

A 37-year-old man comes to the emergency room with fever, chills, and left lower leg pain for 2 days. He was recently discharged from the hospital after arthroscopic knee surgery. Physical examination shows an erythematous lesion with poorly defined margins over the left shin but no fluctuance. Treatment with an intravenous antibiotic is begun. Shortly after starting the infusion, the patient develops flushing, erythema, and pruritus of the upper body. The symptoms resolve after discontinuation of the infusion. Before the next dose with the same agent, the patient is given diphenhydramine and ranitidine and the antibiotic is subsequently given at a slower infusion rate without complications. The patient was most likely treated with an antibiotic that binds to which of the following? This patient's painful erythema with poorly defined margins and fever indicate cellulitis, possibly due to MRSA, given his recent hospital stay. The uneventful administration of the antibiotic following pretreatment with antihistamines and a slower infusion rate indicates that this patient's hypersensitivity reaction of flushing, erythema, and pruritus is not an immune-mediated process. This is consistent with red man syndrome, likely as a result of IV vancomycin administration.

D-alanyl-D-alanine D-alanyl-D-alanine residues are bacterial cell wall precursors and the molecular target of vancomycin. Targeting of this highly conserved sequence is responsible for the broad spectrum of vancomycin activity against gram-positive species (including MRSA), which are the most common cause of cellulitis. Red man syndrome is an infusion rate-dependent effect and is a result of IgE-independent mast cell degranulation.

A 57-year-old woman comes to the physician because of increasing wrinkles on her face and sagging skin. She says that her skin used to be smooth and firm. Examination shows diffuse xerosis and mild atrophy, laxity, and fine wrinkles on the periorbital skin. Which of the following processes is most likely involved in the development of this patient's skin findings? This woman has normal, age-related changes of the skin.

Decrease in elastin fiber assembly Chronological aging of the skin, in particular of the dermis, is due in large part to the decreased assembly of elastin fibers. This accounts for increased skin laxity and rigidity. Atrophy of the dermis due to loss of collagen and volume deficit as a result of reduced glycosaminoglycan synthesis further contribute to the appearance of chronologically aged skin. Moreover, aging skin is prone to xerosis secondary to moisture loss resulting from diminished lipid and sebum production at the surface.

A 49-year-old man comes to the physician because of a 6-hour history of fever, an itchy rash, and generalized body aches. Ten days ago, he received treatment in the emergency department for a snake bite. His temperature is 38.5°C (101.3°F), pulse is 80/min, and blood pressure is 120/84 mm Hg. Physical examination shows multiple well-demarcated, raised, erythematous plaques over the trunk. There is tenderness to palpation and decreased range of motion of the metacarpophalangeal and wrist joints bilaterally. Urine dipstick shows 2+ protein. Further evaluation is most likely to show which of the following? This patient most likely has serum sickness after receiving anti-snake venom. This condition is a type III hypersensitivity reaction that typically manifests with fever, an urticarial rash, myalgia, polyarthritis, and proteinuria 1-2 weeks after exposure to nonhuman protein antigens.

Decreased serum complement concentration Antibody-antigen complexes form after exposure to nonhuman protein antigens (e.g., venom anti-toxins, microbial anti-toxins). If these complexes are inadequately cleared by the mononuclear phagocyte system, they can deposit in tissues and cause serum sickness due to the resulting inflammatory response. This response includes complement system activation and consumption, which results in a decreased serum complement concentration.

A 13-year-old girl is brought to the physician because of an itchy rash on her knee and elbow creases. She has had this rash since early childhood. Physical examination of the affected skin shows crusty erythematous papules with skin thickening. She is prescribed topical pimecrolimus. The beneficial effect of this drug is best explained by inhibition of which of the following processes? This patient's rash is consistent with atopic dermatitis (eczema). Pimecrolimus, which can be used to treat atopic dermatitis, is a calcineurin inhibitor.

Dephosphorylation of serine Calcineurin is a phosphatase that activates the nuclear factor of activated T cell (NFAT) by dephosphorylating its serine and threonine residues. Activation of NFAT induces the transcription of IL-2, which, in turn, activates NK cells and T cells. Immunosuppressants such as pimecrolimus, tacrolimus, and cyclosporine inhibit calcineurin and thereby prevent dephosphorylation of serine on NFAT.

A 34-year-old man comes to the physician because of progressive swelling of the left lower leg for 4 months. One year ago, he had an episode of intermittent fever and tender lymphadenopathy that occurred shortly after he returned from a trip to India and resolved spontaneously. Physical examination shows 4+ nonpitting edema of the left lower leg. His leukocyte count is 8,000/mm3 with 25% eosinophils. A blood smear obtained at night confirms the diagnosis. Treatment with diethylcarbamazine is initiated. Which of the following is the most likely route of transmission of the causal pathogen? This patient's history of tender lymphadenopathy after traveling to India, and his current presentation of unilateral leg swelling, are suspicious for lymphedema in the setting of lymphatic filariasis. The diagnosis is confirmed with a blood smear showing microfilariae, which classically circulate in the bloodstream during the night.

Deposition of thread-like larvae into the skin by a female mosquito Infection with Wuchereria bancrofti causes lymphatic filariasis after a mosquito vector introduces the filarial larvae into the skin during a blood meal. The larvae migrate through the bite wound and enter the lymphatic system, where they mature into adult worms. Infection can cause acute inflammation with symptoms of fever, chills, and tender lymphadenopathy that typically last for 1 week. Then, 9-12 months after the initial infection, chronic inflammation and obstruction of the lymphatic vessels results in lymphedema, as seen in this patient with unilateral lower extremity swelling. First-line treatment for lymphatic filariasis is diethylcarbamazine.

A 35-year-old man comes to the physician because of a rash on the thigh for 10 days. He reports that the rash has been enlarging and is intensely itchy. Two weeks ago, he adopted a stray dog from an animal shelter. Vital signs are within normal limits. A photograph of the examination findings is shown. Which of the following is the most likely cause of this patient's symptoms? The photograph shows an erythematous, annular plaque with central clearing and a raised border.

Dermatophyte infection This patient has a dermatophyte infection, a common fungal infection of the skin, hair, and/or nails transmitted via contact with the skin of an infected person or animal (e.g., this patient's recently adopted dog). Dermatophyte infections can affect the scalp (tinea capitis), inguinal area (tinea cruris), hands (tinea manuum), feet (tinea pedis), nails (tinea unguium), or rest of the body (tinea corporis). Tinea corporis classically spreads centrifugally, manifesting with an erythematous, annular, and pruritic plaque with central clearing and a raised border, as seen in this patient. The diagnosis is confirmed by obtaining scale from a lesion and visualizing branching, septate hyphae on a KOH test. Antifungal treatment options include topical azoles (e.g., ketoconazole) or systemic agents (e.g., oral terbinafine) for extensive and/or refractory disease.

A 17-year-old girl comes to the physician because of a 2-day history of pain in her right knee. Last week she had right wrist pain. She has no history of recent trauma. She returned from summer camp in Connecticut 2 weeks ago. She is sexually active with one male partner and uses an oral contraceptive. Her temperature is 38°C (100.4°F). Examination shows several painless vesiculopustular lesions on the back and one lesion on the right sole of the foot. There is swelling of the right knee with tenderness to palpation. Passive extension of the right wrist and fingers elicits pain. Which of the following is the most likely diagnosis? There are many conditions that can cause both dermatitis and oligoarthritis, but the accompanying fever, painless vesiculopustular lesions, tenosynovitis (pain on passive extension of wrist and fingers), and lack of prior illness or other symptoms may help distinguish between them.

Disseminated gonococcal infection Infection with Neisseria gonorrhoeae is the second most commonly reported infectious disease in the US and can manifest in a localized (e.g., urethritis, pelvic inflammatory disease) or disseminated (arthritis-dermatitis syndrome, purulent gonococcal arthritis) form. The disease is most commonly transmitted during unprotected sexual intercourse. Arthritis-dermatitis syndrome typically presents with migratory, asymmetric arthritis, tenosynovitis, and vesicular, pustular, or maculopapular lesions on the trunk and extremities (sometimes involving the palms and soles), as seen in this patient.

A 20-year-old woman comes to the physician because of a 2-day history of low-grade fever and painful lesions on her left index finger. Two weeks ago, she had a painful rash on the right labia majora that resolved without treatment. Physical examination shows tender lymphadenopathy of the left epitrochlear and right inguinal region. A photograph of the left index finger is shown. Which of the following best describes the properties of the most likely virus involved? This woman has herpetic whitlow, which manifests with a painful, vesicular rash on the fingers. This infection is caused by herpes simplex virus (HSV)-1 or HSV-2. In this patient with a history of a genital rash, HSV-2 is the most likely causal organism.

Enveloped virus with a double-stranded, linear DNA structure The herpes simplex virus is an enveloped virus with a double-stranded, linear DNA structure that causes HSV-1 and HSV-2 infections. The envelope is a lipid bilayer around an icosahedral capsid that contains viral glycoproteins and proteins from the host cell; it helps the virus fuse with and infect host cells. Unlike most other enveloped viruses, which use lipids from host cell plasma membranes to build their envelopes, the herpesviruses use lipids from host cell nuclear membranes. Because immune cells do not typically attack host cell nuclei, the envelope aids in viral evasion of the immune response.

A 17-year-old male comes to the physician because of painful genital sores, malaise, and fever for 3 days. He is sexually active with 3 female partners and does not use condoms consistently. His temperature is 38.3°C (101°F). Physical examination shows tender lymphadenopathy in the left inguinal region and multiple, punched-out ulcers over the penile shaft and glans. Microscopic examination of a smear from the ulcer is most likely to show which of the following? This patient's presentation of tender inguinal lymphadenopathy, multiple punched-out ulcers, and systemic signs of infection (e.g., malaise, fever) suggest herpes genitalis, which is typically caused by herpes simplex virus 2 (HSV-2).

Eosinophilic intranuclear inclusions Eosinophilic intranuclear inclusions on a smear of an ulcer or opened vesicle describe the Cowdry A inclusion bodies seen in infections by herpesviruses such as HSV-1, HSV-2, VZV, and CMV. These intranuclear inclusions are formed during herpesvirus replication within the nucleus. Another microscopic characteristic of herpesvirus infection is the formation of multinucleated giant cells as a result of the fusion of multiple infected cells (seen on Tzanck smear). HSV-2 can remain dormant in the sacral ganglia after the primary infection to cause recurrent genital herpes.

A 5-year-old girl is brought to the physician by her mother because of a 1-month history of a painful ulcer on her face. She has developed painful sunburns in the past with minimal UV exposure. Examination of the skin shows a 2-cm ulcerated nodule on the left cheek. There are scaly, hyperpigmented papules and plaques over the skin of the entire body. Ophthalmologic examination shows decreased visual acuity, clouded corneas, and limbal injection. Examination of a biopsy specimen from the facial lesion shows poorly-differentiated squamous cell carcinoma. Impairment of which of the following proteins is the most likely cause of this patient's condition? This young patient has a history of extreme sensitivity to light and presents with squamous cell carcinoma, several ocular abnormalities, and multiple hyperpigmented patches that likely represent precancerous lesions. These clinical findings are consistent with xeroderma pigmentosum, a rare, autosomal-recessive disease in which the ability to repair UV-induced DNA damage is defective.

Excision endonuclease UV radiation results in dimer formation of neighboring pyrimidine bases, most commonly thymine. These bulky pyrimidine dimers induce helix distortion and can disrupt DNA replication. Dimers are usually repaired by nucleotide excision repair in the G1 phase of the cell cycle, in which the damaged oligonucleotides are first recognized and then excised. The process is carried out by excision endonucleases. DNA polymerase and ligase then fill and reseal the resulting gap. Mutations in any of these endonucleases can lead to extreme UV sensitivity and xeroderma pigmentosum.

A 33-year-old man comes to the physician for evaluation of progressive hair loss from his scalp. He first noticed receding of the hairline over the bitemporal regions of his scalp 5 years ago. Since then, his hair has gradually become thinner over the crown of his head. He is otherwise healthy and takes no medications. Examination shows diffuse, nonscarring hair loss over the scalp with a bitemporal pattern of recession. Administration of which of the following drugs is most appropriate to treat this patient's hair loss? This patient's history of gradual, nonscarring hair loss in a bitemporal pattern is consistent with androgenetic alopecia.

Finasteride Androgenetic alopecia follows an autosomal dominant or polygenic mode of inheritance with variable penetrance and is caused by high serum concentrations of circulating androgens such as dihydrotestosterone. 5-alpha reductase inhibitors (e.g., finasteride) decrease the conversion of testosterone to dihydrotestosterone and are the first-line therapy for men with androgenetic alopecia. Adverse effects include gynecomastia and sexual dysfunction (e.g., decreased libido, erectile dysfunction). Topical minoxidil therapy is a first-line alternative.

A 5-year-old boy is brought to the physician because of a nonpruritic rash on his face that began 5 days ago. It started as a bug bite on his chin that then developed into small pustules with surrounding redness. He has not yet received any routine childhood vaccinations. Physical examination shows small, clustered lesions with gold crusts along the lower lip and chin and submandibular lymphadenopathy. At a follow-up examination 2 weeks later, his serum anti-deoxyribonuclease B antibody titer is elevated. This patient is at greatest risk for which of the following complications? This child's gold-crusted facial lesions are consistent with impetigo, a cutaneous infection that may be caused by Staphylococcus aureus, Streptococcus pyogenes, or coinfection with both pathogens. The elevated anti-deoxyribonuclease B (anti-DNase B) antibody titer indicates that the causal pathogen was S. pyogenes.

Glomerulonephritis Acute post-streptococcal glomerulonephritis (PSGN) is a common complication of impetigo and may occur in response to an infection with nephritogenic strains of group A streptococci. It is caused by immune complex deposition within the glomerular basement membrane, which triggers complement activation and subsequent destruction of the glomeruli (type 3 hypersensitivity reaction). PSGN typically arises 10-30 days following infection, with symptoms such as hematuria, edema, and hypertension. Further complications of group A streptococcal infections include rheumatic fever and scarlet fever. Although scarlet fever occurs in very rare cases after streptococcal skin infections such as impetigo, both conditions are typically only associated with streptococcal tonsillopharyngitis.

A 28-year-old woman comes to the physician because of a 2-month history of multiple right inframammary lumps. They are tender and have a foul-smelling odor. Twelve months ago she had episodes of painful swellings in the axillae that resolved with antibiotic therapy, leaving some scarring. She has Crohn disease. Menses occur at irregular 18- to 40-day intervals and last 1-5 days. The patient's only medication is mesalamine. She appears anxious. She is 162 cm (5 ft 4 in) tall and weighs 87 kg (192 lb); BMI is 33 kg/m2. Vital signs are within normal limits. Examination of the right inframammary fold shows multiple tender, erythematous nodules, and fistulas with purulent discharge. Her fasting glucose concentration is 136 mg/dL. Which of the following areas is also likely affected by this patient's condition? In an obese woman, inflammatory, painful axillary and inframammary lumps associated with scarring and fistulas that drain malodorous pus suggest hidradenitis suppurativa (HS).

Groin Areas of intertriginous folds with hormonally active apocrine sweat glands, such as the groin, axilla, inner thigh, and perineal area, are most commonly involved in patients with HS. HS is a chronic inflammatory condition thought to be caused by blockage of the folliculopilosebaceous unit. The highest incidence of HS is seen in young women. Risk factors such as smoking, obesity, impaired glucose tolerance, inflammatory bowel disease, and a family history of HS are associated with a more severe disease course. Initial treatment consists of general skincare measures (e.g., gentle skin cleansing, wound care), behavioral interventions (e.g., weight loss, smoking cessation), and topical and/or oral antibiotics. If necessary, surgical interventions are performed to drain nodules and excise existing fistulas, scars, and abscesses.

A 23-year-old woman comes to the physician because of a 2-month history of diarrhea, flatulence, and fatigue. She reports having 3-5 episodes of loose stools daily that have an oily appearance. The symptoms are worse after eating. She also complains of an itchy rash on her elbows and knees. A photograph of the rash is shown. Further evaluation of this patient is most likely to show which of the following findings? This woman's rash is consistent with dermatitis herpetiformis and her symptoms of malabsorption (e.g., fatty diarrhea, flatulence, fatigue) are suggestive of celiac disease.

HLA-DQ2 serotype HLA-DQ2 is a cell surface receptor of antigen-presenting cells associated with 90-95% of cases of celiac disease. Expression of HLA-DQ2 is thought to predispose patients to celiac disease due to increased binding of gluten peptides to this HLA subtype, triggering a T-cell mediated autoimmune reaction and inflammation of the intestinal epithelium. Dermatitis herpetiformis is associated with celiac disease and is caused by deposition of IgA and complement C3 in dermal papillae of the dermis.

A 43-year-old woman comes to the physician because of a 1-day history of rash on the trunk and lower extremities. Three days ago, she visited a spa resort with multiple swimming pools and whirlpools. A friend of hers who also visited the spa has developed a similar rash. She does not smoke or drink alcohol and takes no medications. She appears well. Her vital signs are within normal limits. Examination shows multiple erythematous, excoriated papules and pustules over the trunk and upper thighs. The inflammation seen in this patient's condition most likely originated in which of the following parts of the skin? This patient has a self-limited infection that typically manifests 8-48 hours after exposure to water contaminated by Pseudomonas aeruginosa.

Hair follicles Inflammation of the hair follicles is the cause of folliculitis. This patient's history of visiting a spa resort and developing a pruritic, papulopustular rash is highly suggestive of a diagnosis of hot tub folliculitis. This diagnosis is further supported by her friend developing similar symptoms. Though folliculitis is most commonly caused by S. aureus, hot tub folliculitis is characteristically caused by P. aeruginosa, which proliferates in wet and warm environments. This condition is typically self-limited and does not require antibiotic treatment.

A 11-year-old girl comes to the physician for evaluation of recurrent nosebleeds since childhood. She has multiple, small dilated capillaries on the lips, nose, and fingers on exam. Her father has a similar history of recurrent nosebleeds. Which of the following conditions is this patient at increased risk for? This autosomal dominant condition that is causing this patient's telangiectasias is associated with arteriovenous malformations.

High-output heart failure This patient has recurrent episodes of epistaxis and telangiectasias of the lips, nose, and fingers, symptoms that are classic for hereditary hemorrhagic telangiectasia (also known as Osler-Weber-Rendu syndrome). Patients with this syndrome may also present with pulmonary or hepatic arteriovenous shunts, which may lead to high-output cardiac failure. Other complications associated with this syndrome include brain abscesses, stroke due to paradoxical emboli (emboli passing through a pulmonary arteriovenous shunt into systemic circulation, bypassing the lungs), and anemia due to chronic gastrointestinal bleeding from telangiectasias in the GI tract.

A 65-year-old man comes to the physician because he is worried about a mole on his right forearm. He has had the mole for several years, but it has grown in size in the past 3 months. Physical examination shows a hyperpigmented plaque with irregular borders and small area of ulceration. Histopathologic analysis of a full-thickness excisional biopsy confirms the diagnosis of malignant melanoma. Invasion of which of the following layers of skin carries the highest risk of mortality for this patient? Tumor thickness is one of the most important prognostic factors for malignant melanoma.

Hypodermis The hypodermis (subcutis) is the deepest layer of the skin, and melanoma mortality is directly correlated with tumor thickness. Due to variability in the thickness of skin compartments among different individuals, tumor thickness does not always correspond with skin layer invasion. However, in this patient it is likely that a tumor extending into the hypodermis is thicker than a tumor in a more superficial layer. Breslow depth is the most important prognostic factor used for melanoma staging and to determine the safety margin of a full-thickness incision.

A 15-year-old girl is brought to the physician because of a 8-month history of fatigue, intermittent postprandial abdominal bloating and discomfort, foul-smelling, watery diarrhea, and a 7-kg (15-lb) weight loss. She developed a pruritic rash on her knees 3 days ago. Physical examination shows several tense, excoriated vesicles on the knees bilaterally. The abdomen is soft and nontender. Her hemoglobin concentration is 8.2 g/dL and mean corpuscular volume is 76 μm3. Further evaluation of this patient is most likely to show which of the following findings? Postprandial abdominal discomfort and bloating, chronic diarrhea, and severe microcytic anemia (likely secondary to iron deficiency) point to a malabsorptive syndrome. The additional presence of dermatitis herpetiformis (several tense, grouped subepidermal blisters) is strongly suggestive of celiac disease.

IgA tissue transglutaminase antibodies Testing for IgA tissue transglutaminase antibodies is the gold standard for diagnosing celiac disease and is also useful in monitoring a patient's response to treatment, as antibody levels start to decline and usually normalize 3-12 months following the introduction of a gluten-free diet. Although celiac disease can occur at any age, it most commonly manifests either in early childhood or between 20-40 years of age.

A 4-month-old boy is brought to the physician because of a painful lesion on his right leg. Yesterday, he received all of his scheduled childhood immunizations, including the second dose of the diphtheria, tetanus, and acellular pertussis vaccine, which was administered in the right quadriceps. Physical examination shows a 2-cm ulcer with surrounding induration over the right anterolateral thigh. A skin biopsy of the lesion shows localized edema, skin necrosis, neutrophilic infiltration, and complement deposition. Which of the following is the most likely cause of his symptoms? This patient's cutaneous reaction after receiving his second diphtheria and tetanus vaccine is consistent with an Arthus reaction.

Immune complex deposition Immune complex deposition describes the mechanism of Arthus reaction, a localized type III hypersensitivity reaction that can occur 12 - 36 hours after intradermal injection of an antigen in pre-sensitized individuals (circulating antigen-specific IgG). This child was most likely sensitized to an antigen in the DTaP vaccine when he received it for the first time at 2 months of age (routine immunization schedule). DTaP is administered intramuscularly but inadvertent intradermal injection often occurs. The circulating antibodies combine with the antigen to form immune complexes, which trigger localized vasculitis and fibrinoid necrosis by activating the classical complement pathway. An Arthus reaction is usually self-limiting.

A 50-year-old woman comes to the emergency department after waking up with facial swelling and with difficulties swallowing. She was recently diagnosed with hypertension and started on an antihypertensive drug. She follows a strict vegetarian diet. Her pulse is 110/min and blood pressure is 135/85 mm Hg. Physical examination shows marked edematous swelling of the face, lips, and tongue. There is no rash. Serum C4 levels are within the reference range. Which of the following is the most likely cause of this patient's symptoms? This patient's symptoms are concerning for an adverse effect of angiotensin-converting enzyme inhibitors (ACEI).

Impaired breakdown of kinins ACE is one of the enzymes responsible for the degradation of kinins. Therapy with ACEIs can therefore increase serum levels of bradykinin, which in turn can cause angioedema due to vasodilation and increase in vascular permeability. It can also cause dry cough. Severe edema of the tongue and/or larynx can obstruct the upper airway resulting in difficulty swallowing, speaking, and/or stridor. Although these adverse effects typically occur soon after initiation of treatment, they may occur at any time, even years after starting treatment. Unlike angioedema caused by mast-cell degranulation, angioedema caused by an increase in bradykinin is not associated with pruritus or urticaria. ACEIs are the most common cause of nonurticarial angioedema.

A 68-year-old woman is brought to the physician by the staff of a nursing home where she resides because of a painful lesion on her lower back. She has been bedridden since a motorcycle accident in her 50s, which caused a thoracic spine injury. She has type 2 diabetes mellitus treated with metformin. Her temperature is 37.0°C (98.6°F), pulse is 68/min, and blood pressure is 128/74 mm Hg. Cardiopulmonary examination shows no abnormalities. Pulses are palpable in the lower extremities. Neurologic examination shows paraparesis. A photograph of the sacral region is shown. The affected area is not warm but is tender to palpation. Her fasting serum glucose concentration is 97 mg/dL. Which of the following is the most likely explanation for this patient's current condition? The photograph shows a red-purple area of skin over the sacrum, with loss of all skin layers and exposure of the subcutaneous tissue. These findings, in conjunction with this patient's history and clinical presentation, are consistent with a stage 3 decubitus ulcer.

Impaired microcirculation Prolonged external pressure on the skin impairs microcirculation by compressing arterioles, which causes ischemic skin necrosis and decubitus ulcer formation. This patient's paraparesis and immobilization are significant risk factors for decubitus ulcers. Other risk factors include impaired perfusion (e.g., peripheral artery disease), impaired sensation (e.g., neuropathy), and malnutrition. Body areas subjected to higher external pressure are at the greatest risk. Accordingly, pressure ulcers typically occur over bony prominences, such as the sacrum, heel, greater trochanter, lateral malleolus, and elbows. Regularly turning immobilized patients is an effective prevention strategy.

A 32-year-old woman comes to the physician because of a 2-month history of fatigue and yellowish discoloration of her skin. During this period, she has also had a 6-kg (13-lb) weight gain. She does not smoke and drinks one glass of wine daily. She follows a vegan diet that consists mainly of spinach, kale, and sweet potatoes. Physical examination shows yellowing of the skin. Her hands are cold to the touch and her nails appear brittle. Eye examination shows no abnormalities. Which of the following findings is most likely in this patient's serum? Notably, this patient does not present with scleral icterus.

Increased beta-carotene concentration Increased beta-carotene concentration causes pseudojaundice, which manifests with yellow discoloration of the skin, especially in the hands, soles, forehead, and nose because beta-carotene accumulates in the skin's epidermis; the sclerae and mucous membranes are spared, as seen here. Carotenemia can result from excessive consumption of sweet potatoes, leafy greens, carrots, and oranges; increased intake of carotene-rich supplements may also cause carotenemia. This patient's weight gain, fatigue, brittle nails, and cold skin indicates hypothyroidism, which leads to reduced production of vitamin A from beta-carotene due to lack of thyroxine, thereby further increasing beta-carotene levels.

An investigator is conducting a phase 1 trial for a novel epoxide reductase inhibitor with favorable pharmacokinetic properties for cerebrovascular accident prophylaxis. Two days after the trial starts, a subject begins to notice pain and erythema over the right thigh. It rapidly progresses to a purpuric rash with the development of necrotic bullae over the next 24 hours. Laboratory studies show a partial thromboplastin time of 29 seconds, prothrombin time of 28 seconds, and INR of 2.15. Which of the following best describes the pathogenesis of the disease process in the patient? The pharmacodynamic properties of this drug are most similar to warfarin. .

Increased factor VIII activity Epoxide reductase results in the formation of reduced vitamin K, which is essential for activation of both procoagulant factors II, VII, IX, X as well as the anticoagulants protein C and protein S. Of these vitamin K-dependent factors, factors IIa and Xa have long half-lives. Protein C and factor VIIa, on the other hand, have the shortest half-lives and are thus the first components to be depleted following epoxide reductase inhibition. Despite an early increase in PT and INR caused by decreased VIIa (extrinsic pathway factor), the patient is intially in a hypercoagulable state because decreased active protein C levels cause increased activity of VIIIa (intrinsic pathway factor) and Va (common pathway factor). This hypercoagulable state can result in intravascular microthrombosis, which can cut off blood supply to the skin and cause necrosis, as seen in this patient

A 4-year-old boy is brought to the physician by his mother because of painless lesions on his face that he has had since shortly after birth. They recently moved to the USA from Indonesia where they had limited access to healthcare. A photograph of the lesions is shown. Which of the following is the most likely diagnosis? This boy's lesions are the most common benign tumors of infancy. They usually manifest during the first few days to months of life, proliferate rapidly, and then slowly regress spontaneously by 5-9 years of age.

Infantile hemangioma Infantile hemangiomas, also known as strawberry hemangiomas, are benign, vascular tumors that occur in ∼ 10% of infants. These lesions are most commonly found on the head and neck and initially appear as a flat lesion or group of telangiectasias that progresses to a red papule or plaque. Although often solitary, patients can develop multiple lesions as seen in this boy. Histopathology is not necessary for diagnosis but will demonstrate clusters of thin-walled capillaries lined with angioblastic endothelial cells that grow rapidly during the proliferation stage. Treatment is often not required because the lesions are usually asymptomatic and regress spontaneously within years. For rapidly growing lesions or lesions around the eye, therapies include administration of propranolol, cryotherapy, laser ablation, and surgical resection to avoid complications such as ulceration or disfigurement.

A 21-year-old man comes to the physician because of painful, firm, dark bumps on his neck and jawline. He has no history of serious illness and takes no medications. His brother had a similar rash that improved with topical erythromycin therapy. A photograph of the rash is shown. Which of the following is the most likely underlying mechanism of this patient's condition? The image shows multiple small, dark, firm papules and some pustules on a man's shaved neck, which are manifestations of pseudofolliculitis barbae.

Interfollicular penetration of the skin by distal end of hair Pseudofolliculitis barbae primarily affects African American men and is characterized by an inflammatory reaction to ingrown hairs (e.g., after shaving). The hair either enters the interfollicular epidermis after exiting the follicular orifice (extrafollicular penetration) or penetrates the dermis before exiting the follicular orifice (transfollicular penetration). The resulting inflammatory reaction against the hair leads to the characteristic finding of painful papules distributed along the areas of facial hair growth. Management of pseudofolliculitis barbae includes cessation of shaving and adjunctive topical therapies (e.g., topical corticosteroids to reduce inflammation and antimicrobial therapy to treat secondary infection).

A 74-year-old man comes to the physician for evaluation of a skin lesion on his right arm. The lesion first appeared 3 months ago and has since been slowly enlarging. Physical examination shows a 1.5-centimeter, faintly erythematous, raised lesion with irregular borders on the dorsum of the right forearm. A biopsy specimen is obtained. If present, which of the following histopathological features would be most consistent with carcinoma in situ? Neoplasia progresses along a predictable course from normal cellular morphology to the reversible changes of dysplasia and then to carcinoma in situ.

Irreversible nuclear changes in the stratum basale Formation of neoplasia starts with low-grade dysplasia, a reversible disordered growth of epithelium characterized by abnormally frequent mitotic figures, loss of cell orientation and uniformity, and nuclear changes. Carcinoma in situ is usually an irreversible severe (i.e., high-grade) dysplasia involving the entire thickness of the epithelium without penetration of the basement membrane.

An otherwise healthy 66-year-old man comes to the physician for evaluation of rough skin over his forehead and the back of his hands. He has tried applying different types of moisturizers with no improvement. He has worked on a farm all his life. Physical examination shows two erythematous papules with a gritty texture and central scale over the left temple and three similar lesions over the dorsum of his hands. This patient's skin lesions increase his risk of developing a skin condition characterized by which of the following findings on histopathology? This patient has actinic keratoses on his skin associated with his extensive history of occupational sun exposure. These lesions typically manifest as rough, scaly papules on sun-exposed areas and can progress to cutaneous squamous cell carcinoma.

Keratin pearls Keratin pearls are the key histopathologic feature of cutaneous squamous cell carcinoma (SCC). SCC typically manifests as ulcerative, crusted, and/or scaly lesions on sun-exposed areas. It is locally invasive and can spread to the lymph nodes but rarely metastasizes. In addition to extensive sun exposure, SCC of the skin is associated with immunosuppression, chronically draining sinuses, and arsenic exposure.

A 6-year-old boy is brought to the physician because of worsening headaches and a rash for 2 weeks. His mother reports that the rash started on his abdomen and diffusely spread to other areas. Over the past 2 months, he has had recurrent episodes of otitis media. Examination shows a diffuse, erythematous, papular rash involving the groin, abdomen, chest, and back. His cervical lymph nodes are palpable bilaterally. An x-ray of the skull shows well-defined lytic lesions of the left occipital bone and the mastoid bone. Electron microscopy of a biopsy of the patient's posterior cervical lymph nodes shows polygonal cells with organelles shaped like tennis rackets. The cells stain positive for S-100. Clonal proliferation of which of the following types of cells is most likely seen on microscopy? The tennis racket-shaped organelles seen on electron microscopy are Birbeck granules.

Langerhans cells Langerhans cells are macrophages of the skin and lymphoid tissue that present antigens to other cells of the immune system. Clonal proliferation of these cells results in Langerhans cell histiocytosis, a condition that causes lytic skull lesions and a skin rash, as seen in this patient. Recurrent otitis media can occur if the mastoid process is involved. Langerhans cells express CD1a, which mediates antigen presentation, and S-100, which is used as a tumor marker for cells derived from the neural crest.

A 5-year-old boy is brought to the physician by his parents for evaluation of easy bruising. He has met all developmental milestones. Vital signs are within normal limits. He is at the 50th percentile for height and weight. Physical examination shows velvety, fragile skin that can be stretched further than normal and multiple ecchymoses. Joint range of motion is increased. A defect in which of the following is the most likely cause of this patient's condition? This boy has findings consistent with Ehlers-Danlos syndrome, which typically manifests with easy bruising, skin hyperelasticity, and joint hypermobility.

Lysine-hydroxylysine cross-linking Ehlers-Danlos syndrome has several subtypes, all of which share the underlying mechanism of defective collagen synthesis and/or processing. In patients with impaired lysine-hydroxylysine cross-linking of tropocollagen (e.g., due to enzyme deficiencies), stable collagen fibrils cannot be formed, leading to the characteristic presentation seen in this patient. Defects in type III collagen and type V collagen can also cause this condition.

A 76-year-old woman comes to the physician for a 3-day history of swelling and redness of her left lower leg. She has type 2 diabetes mellitus and hypertension. Her temperature is 38.7ºC (101.7ºF). Examination of the left leg shows an erythematous, raised skin lesion with sharply demarcated margins. The affected skin is warm and tender to palpation. A diagnosis of cutaneous streptococcal infection is made. In addition to the upper dermis, the patient's condition is most likely to involve tissue that is derived from which of the following embryological structures? Erysipelas is typically caused by group A streptococci and, as seen here, manifests with an erythematous and sharply demarcated skin lesion and possibly fever. This condition is a superficial skin infection that is limited to the upper dermis and the superficial cutaneous lymphatics.

Mesoderm The mesoderm gives rise to blood vessels and lymphatic vessels, including the superficial cutaneous lymphatics, which are affected in this patient with erysipelas. Other structures that are derived from the mesoderm are bones (except skull bones), muscles, connective tissue, certain organs (e.g., spleen, kidneys, adrenal cortex, gonads), the upper vagina, the serosal linings of body cavities (peritoneum, pericardium, and pleura), the notochord, the pachymeninges (dura mater), and microglial cells.

A 10-year-old boy is brought to the physician because of a generalized, non-itchy rash for 2 days. He has also had a fever and sore throat for 4 days. He has not received any routine childhood vaccinations. Examination shows a flushed face except around his mouth. A diffuse, papular, erythematous rash that blanches with pressure is seen over the trunk. Pharyngeal erythema and a red beefy tongue are noted. His leukocyte count is 11,200/mm3 (75% segmented neutrophils, 22% lymphocytes). Without treatment, this patient's current condition puts him at greatest risk for which of the following complications? Sore throat and fever followed by a diffuse macular rash, an erythematous tongue (strawberry tongue), and perioral pallor are indicative of scarlet fever.

Mitral valve regurgitation Mitral valve regurgitation (MR) is a complication of untreated group A streptococcal (GAS) infections such a scarlet fever and tonsillopharyngitis. MR is the result of rheumatic heart disease, which can cause early mitral regurgitation or prolapse and later evolves into mitral stenosis.

A 5-year-old boy is brought to the physician because of a painful, burning rash on his left arm for 3 days. Three years ago, he was diagnosed with heart failure due to congenital heart disease and received an allogeneic heart transplantation. He takes cyclosporine to prevent chronic transplant rejection. He has not received any routine childhood vaccinations. A photograph of the rash is shown. Microscopic examination of a skin biopsy specimen is most likely to show which of the following findings? This boy has a painful rash of grouped vesicles on an erythematous base in a dermatomal distribution (C7-C8), which is characteristic of shingles.

Multinucleated epidermal giant cells and intranuclear inclusions Multinucleated giant cells and Cowdry A inclusions are characteristic histopathologic findings found on a Tzanck test of shingles lesions. Initial infection with varicella zoster virus (VZV) causes chickenpox, which this patient likely developed because he has not received any vaccinations. Immunosuppression (e.g., from cyclosporine taken to prevent chronic transplant rejection) is a risk factor for reactivation of VZV (shingles) after a period of latency in dorsal root ganglia. Other risk factors include older age, HIV infection, and malnutrition.

hyperpigmented lesions on the dorsum of both hands. The causal pathogen of this patient's underlying condition was most likely acquired in which of the following ways? This patient's scleral icterus and jaundice should raise suspicion for liver disease. His pruritic rash that worsens upon exposure to sunlight is suggestive of porphyria cutanea tarda, a condition that is associated with hepatitis C virus infection.

Needlestick injury Needlestick injury is a likely cause of this patient's hepatitis C infection as it is mainly transmitted parenterally. Therefore, certain populations such as health care workers and intravenous drug users are at increased risk of contracting the virus. Patients who received blood transfusions before 1992 are also at increased risk for hepatitis C, as donor blood was not screened for the virus before 1992. Other routes of transmission include vertical (from mother to fetus) and, rarely, sexual. Inhalatio

A 59-year-old man comes to the physician for evaluation of a progressively enlarging, 8-mm skin lesion on the right shoulder that developed 1 month ago. The patient has a light-skinned complexion and has had several dysplastic nevi removed in the past. A photograph of the lesion is shown. The lesion is most likely derived from cells that are also the embryological origin of which of the following tumors? The lesion in the photograph is asymmetric, irregularly bordered, variegated in color, > 6 mm in diameter, and developed recently. All of these features, in combination with a history of dysplastic nevi, are concerning for melanoma, which is a malignant tumor of melanocytes. These cells originate from neural crest cells.

Neuroblastoma Neuroblastomas are tumors derived from chromaffin cells of the adrenal medulla that, like melanomas, originate from neural crest cells. Other notable derivatives of neural crest cells include Schwann cells, facial cartilage, the aorticopulmonary septum, and odontoblasts.

A 4-month-old boy is brought to the physician by his father because of a progressively worsening rash on his buttocks for the last week. He cries during diaper changes and is more fussy than usual. Physical examination of the boy shows erythematous papules and plaques in the bilateral inguinal creases, on the scrotum, and in the gluteal cleft. Small areas of maceration are also present. A diagnosis is made, and treatment with topical clotrimazole is initiated. Microscopic examination of skin scrapings from this patient's rash is most likely to show which of the following findings? This patient has typical signs of a diaper rash due to Candida albicans. It can be treated with a variety of topical antifungal agents such as clotrimazole.

Oval, budding yeast with pseudohyphae Candida albicans appears as oval, budding yeast with pseudohyphae in skin scrapings of an infection. Candida albicans is part of the normal skin flora, but infection can occur when there is an imbalance that allows local Candida albicans overgrowth. Infants are often affected by Candida mucocutaneous infections in the anogenital region since overgrowth is facilitated by the prolonged damp and warm conditions of a diaper.

A 51-year-old woman comes to the emergency department because of a 1-day history of severe pain in her left knee. To lose weight, she recently started jogging for 30 minutes a few times per week. She has type 2 diabetes mellitus and hypertension treated with metformin and chlorothiazide. Her sister has rheumatoid arthritis. She is sexually active with two partners and uses condoms inconsistently. On examination, her temperature is 38.5°C (101.3°F), pulse is 88/min, and blood pressure is 138/87 mm Hg. The left knee is swollen and tender to palpation with a significantly impaired range of motion. A 1.5-cm, painless ulcer is seen on the plantar surface of the left foot. Which of the following is most likely to help establish the diagnosis? Permanent joint destruction is a dreaded complication of this patient's condition.

Perform arthrocentesis Arthrocentesis is the best next step in this patient with typical symptoms of septic arthritis, including fever, joint pain, and restricted range of motion. The knee is a commonly affected joint in septic arthritis, which is most commonly caused by hematogenous spread of bacteria (most commonly Staphylococcus aureus) from a distant wound site, such as an infected diabetic ulcer. Typical findings in arthrocentesis include a synovial fluid WBC count of over 50,000/μL with dominance of polymorphonuclear cells (PMN).

An otherwise healthy 15-year-old girl is brought to the physician for evaluation of severe acne that involves her face, chest, and back. It has not improved with her current combination therapy of oral cephalexin and topical benzoyl peroxide. She is sexually active with one male partner, and they use condoms consistently. Facial scarring and numerous comedones are present, with sebaceous skin lesions on her face, chest, and back. Before initiating treatment, which of the following is the most appropriate next step? Treatment with oral retinoids (e.g., isotretinoin) is indicated for forms of (severe) acne vulgaris that have not been responsive to treatment with a combination of oral antibiotics and topical agents. However, systemic retinoid therapy can have severe side effects.

Perform quantitative beta-hCG assay Since retinoids are extremely teratogenic, all female patients must undergo a serum pregnancy test that quantitatively measures hCG levels or a urine pregnancy test that measures these qualitatively prior to initiation of therapy. The pregnancy test should be repeated monthly up to one month after ceasing systemic retinoid therapy. Meanwhile, the patient should commit to a "double" contraception strategy, a combination of an oral as well as another contraceptive (e.g., barrier contraception, IUD). Contraception should start one month before isotretinoin therapy and be continued until one month after isotretinoin therapy is completed. Since the use of a retinoid is frequently associated with decreased liver function and hyperlipidemia, regular liver function and lipid profile tests (both before and during therapy) are recommended.

A 16-year-old boy is brought to the physician for a follow-up appointment. He has a seizure disorder treated with valproic acid. He has always had difficulties with his schoolwork. He was able to walk independently at the age of 2 years and was able to use a fork and spoon at the age of 3 years. Ophthalmic examination shows hyperpigmented iris nodules bilaterally. A photograph of his skin examination findings is shown. This patient is at increased risk for which of the following conditions? This boy presents with a seizure disorder, intellectual disability, developmental delay (walking well and using a fork and spoon should occur at ∼ 14-15 months), café au lait spots (hyperpigmented macules), and hyperpigmented iris (Lisch) nodules, all of which are consistent with neurofibromatosis type 1 (NF1). .

Pheochromocytoma Pheochromocytomas are classically associated with neurofibromatosis type 1. Patients with NF1 are also at increased risk for optic gliomas, especially as children. Hereditary pheochromocytomas are also associated with von Hippel-Lindau disease and multiple endocrine neoplasia type 2 (MEN 2A, MEN 2B)

Four days after being admitted to the hospital for widespread second-degree burns over his arms and thorax, a 29-year-old man develops a fever and wound discharge. His temperature is 38.8°C (101.8°F). Examination shows a discolored burn eschar with edema and redness of the surrounding skin. The wounds have a sickly, sweet odor. A culture of the affected tissue grows an aerobic, gram-negative rod. The causal pathogen most likely produces which of the following substances? This patient has developed a secondary infection of his burn wounds while in the hospital. The characteristic sweet odor and the aerobic, gram-negative rod on culture make Pseudomonas aeruginosa the most likely causal organism.

Phospholipase C Phospholipase C, which degrades cellular membrane phospholipids, is one of several toxins produced by Pseudomonas aeruginosa. This pathogen is one of the organisms that most frequently cause wound infection and sepsis in burn patients. Because burns damage the physical barrier, impair perfusion, and can suppress the immune system, they predispose to infection.

A 14-year-old boy comes to the physician because of an itchy rash on his right arm for 1 day. The rash started as small papules, then progressed into blisters with oozing. He has had atopic dermatitis since the age of 6 years. His vital signs are within normal limits. A photograph of the patient's arm is shown. There is no lymphadenopathy. Avoidance of contact with which of the following would most likely have prevented this patient's symptoms? This patient's symptoms are likely the result of a T cell-mediated hypersensitivity reaction.

Plants This patient's pruritic and erythematous papular rash with linear streaks of small oozing vesicles should raise suspicion for allergic contact dermatitis (ACD), a type IV hypersensitivity reaction. The characteristic skin lesions commonly appear 12-48 hours after exposure to the allergen in individuals that have previously been sensitized. Triggers of ACD include certain plants (i.e., poison oak, poison ivy, or poison sumac, which cause urushiol-induced dermatitis), nickel, rubber/latex, cosmetics, and certain topical medications. Management consists of avoidance of triggers, topical corticosteroids, and wet dressings for mild to moderate cases, and systemic corticosteroids for severe cases (e.g., widespread rash, ACD involving the face).

A 58-year-old man comes to the physician because of a 3-month history of diffuse muscle pain, malaise, pain in both knees, recurrent episodes of abdominal and chest pain. He has also had a 5-kg (11-lb) weight loss over the past 4 months. Four years ago, he was diagnosed with chronic hepatitis B infection and was started on tenofovir. There are several ulcerations around the ankle and calves bilaterally. Perinuclear anti-neutrophil cytoplasmic antibodies are negative. Urinalysis shows proteinuria and hematuria. Muscle biopsy shows a transmural inflammation of the arterial wall with leukocytic infiltration and fibrinoid necrosis. Which of the following is the most likely diagnosis? This patient's presentation with several nonspecific symptoms (arthralgia, malaise, weight loss), cutaneous symptoms (several ulcerations), cardiac symptoms (chest pain), gastrointestinal symptoms (abdominal pain), and renal symptoms (proteinuria, hematuria) is consistent with a multisystemic vasculitis. His negative p-ANCA, hepatitis B infection, and biopsy findings indicate a certain subtype of vasculitis that usually spares the lungs.

Polyarteritis nodosa Polyarteritis nodosa (PAN) like most vasculitides, usually involves multiple organ systems, as seen in this patient (skin, kidneys, heart). The absence of p-ANCA and pulmonary symptoms, as well as the association with Hepatitis B (and C), helps to differentiate PAN from other forms of vasculitis. Muscle biopsy typically shows transmural inflammation of the arterial wall with leukocytic infiltration and fibrinoid necrosis, as seen in this patient. Other typical characteristics of PAN include a high ESR level, proteinuria, and hematuria. PAN should be always considered in young adults presenting with stroke or myocardial infarction.

A 23-year-old woman comes to physician for an annual health maintenance examination. She feels well. She is 155 cm (5 ft 1 in) tall and weighs 79 kg (174 lb); BMI is 33 kg/m2. Examination shows a skin rash over both axillae. A photograph of her left axilla is shown. This patient's skin finding is most likely associated with which of the following conditions? The picture shows acanthosis nigricans, a skin condition that manifests with thickened, velvety, hyperpigmented plaques and most frequently involves intertriginous sites such as the axillae or the neck.

Polycystic ovarian syndrome Acanthosis nigricans is commonly seen in disorders characterized by insulin resistance and hyperinsulinemia (e.g., polycystic ovarian syndrome, obesity, diabetes mellitus, acromegaly). Elevated levels of insulin stimulate keratinocyte and dermal fibroblast proliferation via interaction with insulin-like growth factor 1, resulting in epidermal hyperplasia and plaque-like lesions, as seen in this patient. In addition, insulin resistance in PCOS is responsible for hyperandrogenism (leading to acne, hirsutism) and anovulation (causing amenorrhea and impaired fertility).

A 4-year-old girl is brought to the physician for evaluation of a rash that her mother noticed 5 months ago. The rash is not painful or itchy but she notices that her daughter sometimes picks at the "spots." The girl's 2-year-old brother has also started developing similar skin lesions over the past month. The patient has no history of serious illness and takes no medications. She is in the 75th percentile for height and 50th percentile for weight. A photograph of the patient's rash is shown. Which of the following infectious agents is the most likely cause of this patient's skin condition? Histological evaluation of the skin lesions would show keratinocytes with eosinophilic intracytoplasmic inclusion bodies, also known as molluscum bodies.

Poxvirus The molluscum contagiosum virus is a DNA poxvirus that typically causes multiple nontender, skin-colored, pearly, dome-shaped papules with central umbilication, which can be located on the face, trunk, and extremities, as seen in this patient. Molluscum contagiosum most commonly affects children under 5 years of age. The virus is transmitted through direct skin contact, which explains why the patient's younger brother is also infected. Typically, the lesions are self-limiting within a few months.

A 3-year-old boy is brought to the emergency department after the sudden onset of a rash that started on the head and progressed to the trunk and extremities. Over the past week, he has had a runny nose, a cough, and red, crusty eyes. He recently immigrated with his family from Yemen and immunization records are unavailable. The patient appears malnourished. His temperature is 40.0°C (104°F). Examination shows generalized lymphadenopathy and a blanching, partially confluent maculopapular exanthema. Administration of which of the following is most likely to improve this patient's condition? Coryza, cough, and conjunctivitis followed by maculopapular, blanching, partially confluent exanthema are characteristic of measles.

Retinol While there is no curative treatment for measles, vitamin A (retinol) should be administered to all children with acute measles (esp. important in those with signs of malnutrition) because it helps prevent complications such as otitis media, viral pneumonia, encephalitis, and subacute sclerosing panencephalitis. Vitamin A enhances the barrier function and immunity of the ocular, respiratory, and gastrointestinal epithelium. Lower serum levels of vitamin A are associated with a more severe and prolonged course of the disease.

A 6-year-old boy is brought to the physician for evaluation of bilateral erythematous skin lesions on the flexures of the elbows and knees. He has been scratching the areas frequently. He has had similar lesions intermittently for the last 2 years. A photograph of the lesions on the back of the knees is shown. This patient's skin lesions are most likely associated with which of the following? This patient's highly pruritic erythematous papules, confluent scaly macules, and lichenified plaques on the popliteal and antecubital fossae with a relapsing, chronic course are highly suggestive of atopic dermatitis.

Reversible decrease in FEV1/FVC ratio A reversible decrease in FEV1/FVC ratio with the administration of bronchodilators is a characteristic spirometry finding in asthma. Atopic dermatitis, as seen in this patient, is often associated with other atopic diseases such as asthma and allergic rhinitis (called the "atopic triad" if all three conditions are present).

A previously healthy 5-year-old boy is brought to the physician because of increasing weakness and a retroauricular rash that started 2 days ago. The rash spread rapidly and involves the trunk and extremities. Last week, he had a mild sore throat, pink eyes, and a headache. His family recently immigrated from Ethiopia. His immunization status is unknown. The patient appears severely ill. His temperature is 38.5°C (101.3°F). Examination shows tender postauricular and suboccipital lymphadenopathy. There is a nonconfluent, maculopapular rash over the torso and extremities. Infection with which of the following is the most likely cause of this patient's symptoms? This young boy presents with a history of prodromal symptoms (sore throat, pink eyes, headache) and a rash behind the ears that quickly developed into a generalized maculopapular exanthem. The overall presentation, in addition to postauricular and suboccipital lymphadenopathy and missing vaccination records, is concerning for rubella infection.

Rubella virus Rubella is caused by the rubella virus, an RNA virus of the genus Rubivirus (formerly part of the family Togaviridae) and family Matonaviridae. In addition to the exanthem and prodromal symptoms seen in this boy, some patients also have a maculopapular enanthem on the soft palate (Forchheimer sign). Teenagers and adults with rubella often present with polyarthritis as well. Although the infection usually has a benign course and is self-limited, infection during pregnancy can lead to congenital rubella syndrome in the neonate, manifesting with cataracts, sensorineural hearing loss, and heart defects.

A 42-year-old man comes to the physician with a 3-month history of an itchy rash on his face. Six months ago, he was diagnosed with early-onset Parkinson disease. His only medication is pramipexole. Physical examination shows the findings seen in the photograph. There is copious dandruff on his scalp without underlying erythema. Which of the following is the most likely diagnosis? This is a benign condition with a biphasic age of onset that most commonly manifests with scales on the scalp in infancy (cradle cap) or on the face in adolescence or early adulthood.

Seborrheic dermatitis This patient's pruritic rash affecting areas of high sebaceous activity (i.e., face, eyelids, scalp) is typical of seborrheic dermatitis. While most cases are idiopathic, there is an increased prevalence of seborrheic dermatitis in patients with HIV infection, underlying neurological conditions (e.g. Parkinson disease, stroke), and intestinal malabsorption syndromes (e.g., celiac sprue). Malassezia furfur and Malassezia globosa are yeasts that are thought to play a role in the pathogenesis of seborrheic dermatitis, but the exact mechanism is unknown. Scalp lesions are typically treated with antifungal shampoos, and facial lesions are treated with low-dose topical steroids and/or antifungal creams.

A 34-year-old man comes to the physician for evaluation of a rash on the elbows for several months. A biopsy of the affected area shows a thinned stratum granulosum as well as retained nuclei and spongiotic clusters of neutrophils in the stratum corneum. This patient's skin findings are most likely associated with which of the following conditions? Thinning of the stratum granulosum, parakeratotic stratum corneum (retained nuclei), and clusters of neutrophils in the stratum corneum (Munro microabscesses) on this patient's skin biopsy are highly suggestive of psoriasis.

Seronegative spondylarthropathy Psoriatic arthritis is a seronegative spondyloarthropathy seen in up to ⅓ of patients with psoriasis. Apart from skin involvement, common findings in psoriatic arthritis include asymmetric arthritis, nail lesions, inflamed, sausage-like fingers (dactylitis), and pencil-in-cup deformity of the distal interphalangeal joints (DIPs) on x-ray.

A 23-year-old woman comes to the physician because she is embarrassed about the appearance of her nails. She has no history of serious illness and takes no medications. She appears well. A photograph of the nails is shown. Which of the following additional findings is most likely in this patient? The photograph shows small, round depressions in the nail (pitting), which is seen in psoriasis.

Silvery plaques on extensor surfaces Silvery plaques on the extensor surfaces are the characteristic skin finding seen in psoriasis, which can also cause nail pitting. Nail pitting results from focal areas of abnormal keratinization of the nail matrix due to increased proliferation of keratinocytes. As is the case here, the pits are usually deep, irregular, and randomly distributed.

A 76-year-old man with a history of type 2 diabetes mellitus comes to the physician for a follow-up examination. He feels well overall but is concerned about some lesions on his neck that his wife noticed a while ago. He is 165 cm (5 ft 5 cm) tall and weighs 84 kg (185 lb); BMI is 31 kg/m2. Physical examination shows lesions scattered across his neck and axilla. A photograph of the lesions on his neck is shown. Which of the following is the most likely diagnosis? The multiple pedunculate appendices with narrow stalks are benign outgrowths of normal skin.

Skin tags Skin tags (acrochordons) are benign outgrowths of normal skin that appear as pedunculated appendices with narrow stalks. The prevalence is 50-60% of individuals aged over 50 years and increases with age. The etiology of skin tags is not fully understood, but they tend to originate on areas of the body that experience friction, such as the lower neck, axilla, inframammary fold, and inguinal region. Skin tags are also associated with diabetes and obesity and may be a sign of insulin resistance and metabolic syndrome. Because skin tags are benign lesions, intervention is typically not necessary, but they may be removed for cosmetic purposes using fine-grade scissors, cryosurgery, or electrodesiccation.

A previously healthy 10-year-old girl is brought to the physician because of severe malaise, pink eyes, cough, and a runny nose for 3 days. She recently immigrated from Sudan and immunization records are unavailable. Her temperature is 40.1°C (104.1°F). Examination shows bilateral conjunctival injections. There are multiple bluish-gray lesions on an erythematous buccal mucosa and soft palate. This patient is at increased risk for which of the following complications? This young girl has a 3-day history of coryza, cough, conjunctivitis (the "3 Cs"), fever, and multiple bluish-gray lesions on an erythematous background (suggestive of Koplik spots). She probably did not receive the recommended vaccinations, which, in combination with her presentation, suggests an active measles infection in the prodromal (catarrhal) stage

Subacute sclerosing panencephalitis Subacute sclerosing panencephalitis is a rare and currently untreatable complication of measles infection that usually develops many years after the initial infection. Although the exact etiology of SSPE is not fully understood, it is thought to result from persistent replication of a defective variant of the measles virus within neurons and glial cells. Particles of this defective virus accumulate intracellularly and are detectable as inclusion bodies on brain tissue analysis. These particles cause inflammation of the parenchyma and ultimately lead to neuronal degeneration, gliosis, and demyelination. Cerebrospinal fluid analysis shows high levels of anti-measles antibodies.

A 36-year-old man comes to the physician because of a 2-day history of malaise and a painful, pruritic rash on his lower back and thighs. His temperature is 37.8°C (100°F). Physical examination shows the findings in the photograph. Skin scrapings from the thigh grow neutral colonies on MacConkey agar. The colony-producing bacteria are oxidase-positive. Which of the following is the greatest risk factor for the patient's condition? MacConkey agar only allows growth of gram-negative organisms. While lactose-fermenting bacteria produce bright pink colonies on this agar, lactose nonfermenters grow in neutral colors. This patient's papulopustular skin eruptions are most likely caused by Pseudomonas aeruginosa, a gram-negative, lactose non-fermenting, oxidase-positive bacillus.

Swimming in pool This patient's clinical symptoms (mild fever, pruritic rash, and skin eruptions) are classical findings of hot tub folliculitis, an infection caused by Pseudomonas aeruginosa. While typically dependent on direct inoculation (such as nail puncture wounds) or an immunocompromised host to cause an infection, the most common route of pseudomonal infection in immunocompetent hosts is water (hot tub, lake, pool). Pseudomonal folliculitis is a self-limiting condition, but antibiotic treatment may be warranted in severe cases.

A 59-year-old man comes to the physician because of a 4-month history of a pruritic rash. His symptoms have not improved despite treatment with over-the-counter creams. During this period, he has also had a 6-kg (13.5-lb) weight loss. Examination shows a scaly rash over his chest, back, and thighs. A photograph of the rash on his thighs is shown. A biopsy of the skin lesions shows clusters of neoplastic cells with cerebriform nucleiA 59-year-old man comes to the physician because of a 4-month history of a pruritic rash. His symptoms have not improved despite treatment with over-the-counter creams. During this period, he has also had a 6-kg (13.5-lb) weight loss. Examination shows a scaly rash over his chest, back, and thighs. A photograph of the rash on his thighs is shown. A biopsy of the skin lesions shows clusters of neoplastic cells with cerebriform nuclei within the epidermis. This patient's condition is most likely caused by the abnormal proliferation of which of the following cell types? The clusters of intraepidermal neoplastic cell aggregates with cerebriform nuclei are known as Pautrier microabscesses.

T cells Proliferation of neoplastic CD4+ T cells with distinctive cerebriform nuclei is the hallmark of mycosis fungoides. Malignant CD4+ T cells aggregate within the epidermis, where they form Pautrier microabscesses. Mycosis fungoides is a chronic, low-grade, cutaneous non-Hodgkin lymphoma characterized by scaly, pruritic, well-demarcated skin plaques and patches. In the later stages of the disease, mushroom-shaped tumors develop within the plaque lesions. Leukemic dissemination of the neoplastic CD4+ T cells results in Sézary syndrome.

A 21-year-old woman comes to the physician because of a 1-week history of white discoloration of the tongue. She has had similar, recurrent episodes over the past 5 years. Examination shows white plaques on the tongue that easily scrape off and thick, cracked fingernails with white discoloration. KOH preparation of a tongue scraping shows budding yeasts with pseudohyphae. This patient's condition is most likely caused by decreased activity of which of the following? A KOH preparation of a tongue scraping that shows budding yeasts with pseudohyphae confirms the diagnosis of oral candidiasis (thrush). This woman's nail findings are suspicious for onychomycosis. Chronic and/or recurrent, localized candidal infections of the oral cavity and nails are common features of chronic mucocutaneous candidiasis.

T cells T cells play a key role in immunity against fungi and viruses. T cell dysfunction is responsible for a variety of immune deficiencies, including chronic mucocutaneous candidiasis (CMCC), which typically presents with noninvasive Candida albicans infections of the skin, mucous membranes, and nails. Examples of congenital T-cell immunodeficiencies include DiGeorge syndrome, Hyper IgE syndrome, IL-12 deficiency, severe combined immunodeficiency, and Wiskott-Aldrich syndrome. CMCC with features of autoimmune disease is typically caused by a deficiency in the AIRE protein, IL-17, or IL-17 receptors.

A 48-year-old man comes to the physician for evaluation of an intensely pruritic skin rash on his arms and legs for 12 hours. Two days ago, he returned from an annual camping trip with his son. The patient takes no medications. A photograph of the skin lesions on his left hand is shown. Activation of which of the following cell types is the most likely cause of this patient's skin findings? The onset of pruritic vesicles arranged in a linear pattern shortly after outdoor activities suggests allergic contact dermatitis, a type IV hypersensitivity reaction, e.g., due to contact with urushiol-containing plants such as poison oak, poison ivy, or poison sumac.

T cells This patient's rash suggests a delayed-type hypersensitivity reaction (type IV hypersensitivity reaction) caused by exposure to allergens (e.g., poison ivy). Antigens penetrating the skin are phagocytosed by Langerhans cells, which subsequently migrate to the lymph nodes and there induce the formation of sensitized T lymphocytes. On a subsequent exposure, these sensitized T lymphocytes secrete lymphokines (e.g., interferon-γ) that induce macrophage-mediated tissue inflammation as seen in this patient. Examples of diagnostic tests that evaluate for a type IV hypersensitivity reaction include tuberculin skin test, Candida extract test, and patch test.

Two weeks after undergoing an allogeneic skin graft procedure for extensive full-thickness burns involving his left leg, a 41-year-old man develops redness and swelling over the graft site. He has not had any fevers or chills. His temperature is 36°C (96.8°F). Physical examination of the left lower leg shows well-demarcated erythema and edema around the skin graft site. The graft site is minimally tender and there is no exudate. Which of the following is the most likely underlying mechanism of this patient's skin condition? Pain and edema within 6 months of receiving an allogeneic skin graft are signs of acute graft rejection.

Th1-induced macrophage activation Th1 cell-induced macrophage activation is responsible for this patient's acute graft rejection. This type IV hypersensitivity reaction is triggered by foreign antigens from the allogeneic skin graft, which incite host T-cell sensitization. Continued exposure to the foreign antigen causes the pre-sensitized T cells to activate macrophages and thus induce an inflammatory reaction in the tissue. In addition, this exposure leads to antibody-mediated complement activation (similar to hyperacute transplant rejection), during which IgM and IgG antibodies develop in response to foreign antigens from an allogeneic graft. This reaction most commonly presents as redness, swelling, itching, and tissue sloughing at the graft site.

A 21-year-old woman comes to the physician because of hair loss on her frontal scalp over the past year. Menses have occurred at irregular 40- to 60-day intervals since menarche at the age of 17 years. She has no history of serious illness and takes no medications. She is 162 cm (5 ft 3 in) tall and weighs 73 kg (158.7 lb); BMI is 28 kg/m2. Her pulse is 75/min and blood pressure 130/76 mm Hg. Physical examination shows scattered pustules on her face and patches of velvety hyperpigmentation on her axilla and groin. Her morning serum cortisol concentration is 18 μg/dL. This patient's condition is most likely associated with increased stimulation of which of the following types of cells? Frontal hair loss, menstrual irregularities, overweight, acne vulgaris (scattered facial pustules), and acanthosis nigricans (velvety hyperpigmentation on axilla and groin) suggest polycystic ovary syndrome.

Theca interna cells Polycystic ovary syndrome (PCOS) is characterized by an increase in the secretion of pituitary luteinizing hormone (LH), which stimulates ovarian theca interna cells to produce androgens. The resulting hyperandrogenism accounts for the typical clinical features of PCOS: cutaneous manifestations (hirsutism, androgenic alopecia, and acne vulgaris), anovulation/oligoovulation (often resulting in infertility), and polycystic ovaries on ultrasound. A majority of affected women also have hyperinsulinism and/or insulin resistance, leading to symptoms such as acanthosis nigricans and overweight, which are seen in this patient.

A 35-year-old man comes to the physician because of an ulcer on his penis that he first noticed 4 days ago. He is currently sexually active with multiple male partners and uses condoms inconsistently. Genital examination shows a shallow, nontender ulcer with a smooth base and indurated border along the shaft of the penis. There is bilateral inguinal lymphadenopathy. Darkfield microscopy of a sample from the lesion shows gram-negative, spiral-shaped bacteria. A drug that acts by inhibition of which of the following is the most appropriate treatment for this patient? A nontender genital ulcer (chancre) with associated inguinal lymphadenopathy suggests a diagnosis of primary syphilis. The presence of spirochetes on darkfield microscopy confirms infection with Treponema pallidum, the causal pathogen of syphilis. The first-line therapy for this condition is intramuscular benzathine penicillin G.

Transpeptidase Penicillin G, a beta-lactam antibiotic, binds to penicillin-binding proteins (transpeptidase) and inhibits cross-linking of peptidoglycan during bacterial cell wall synthesis. It has antimicrobial activity against streptococci, pneumococci, meningococci, and Treponema pallidum.

A 26-year-old man is brought to the emergency department because of abdominal pain, dizziness, shortness of breath, and swelling and pruritus of the lips, tongue, and throat for 1 hour. The symptoms began minutes after he started eating a lobster dinner. It is determined that his symptoms are due to surface crosslinking of IgE. This immunologic event most likely caused the release of which of the following? The patient's acute-onset abdominal pain, dizziness, shortness of breath, swelling, and pruritus after eating shellfish (a common allergen) are characteristic of an anaphylactic reaction. Anaphylaxis is a type I hypersensitivity reaction mediated by surface crosslinking of IgE on mast cells.

Tryptase In the immediate phase of anaphylactic reactions, surface crosslinking of IgE causes mast cell degranulation, which releases tryptase, histamine, heparin, and eosinophil chemotactic factors. Of these compounds, tryptase is a relatively specific marker of mast cell activation.

A 28-year-old African American woman delivers her first child at 39 weeks' gestation. On examination, the newborn's skin and hair have a milky white color, and his eyes are light blue with a slight reddish hue. A skin biopsy shows a normal number of epidermal and follicular melanocytes. The physician advises the parents that the child will need a comprehensive eye examination and genetic testing to establish the suspected diagnosis. A defect in which of the following enzymes is the most likely cause of this patient's condition? Dihydroxyphenalanine (DOPA), an intermediate compound in the synthesis of catecholamines, serves as one of the substrates of this enzyme. A deficiency of this enzyme will result in oculocutaneous albinism, as seen in this patient.

Tyrosinase Tyrosinase is a copper-containing enzyme in melanocytes that produces melanin from dihydroxyphenylalanine (DOPA), which is derived from tyrosine by the action of tyrosine hydroxylase or tyrosinase itself. Tyrosinase is encoded by the TYR gene on chromosome 11; a defect in this gene is responsible for oculocutaneous albinism type 1 (OCA1), which manifests as hypopigmentation of the hair, skin, and irides with a normal density of melanocytes on skin biopsy, as seen here. The neurological and physical development of infants with OCA1 is similar to that of normal infants, but they have an increased risk of skin cancer and visual abnormalities and therefore need to undergo skin and eye screenings periodically.

nd swelling of both legs. The symptoms are worst at the end of the day and are associated with itching of the overlying skin. Physical examination shows bilateral pitting ankle edema. An image of one of the ankles is shown. This patient is at greatest risk for which of the following complications? The varicose veins, brown skin discoloration, and corona phlebectatica in this patient are signs of chronic venous insufficiency.

Ulceration of the skin In chronic venous insufficiency, defective venous valve leaflets lead to reflux of venous blood back into the periphery, resulting in increased venous pressure (hydrostatic pressure) and subsequent extravasation of venous blood into the interstitium. Plasma proteins in the interstitium, in turn, cause edema and tissue hypoperfusion, which results in stasis dermatitis, lipodermatosclerosis, and, in some cases, ulcer formation. Venous ulcers tend to occur over the shin and medial malleolus and are shallow, tender, and surrounded by hyperpigmented skin.

A 31-year-old male comes to the physician because of a 2-day history of blisters and brownish discoloration of urine. His symptoms appeared after he returned from a 4-day trip with his friends in Florida. He has had similar episodes of blistering twice in the past three years. Each episode resolved spontaneously after a few weeks. Examination shows vesicles and bullae on the face and the dorsal surfaces of his hands and forearms. His condition is most likely caused by a defect in which of the following enzymes? This patient with recurrent blisters on the face and dorsum of hands and forearms and dark urine in the setting of sunlight exposure likely has porphyria cutanea tarda.

Uroporphyrinogen III decarboxylase Insufficiency in uroporphyrinogen III decarboxylase leads to porphyria cutanea tarda. The blistering pattern follows what would be expected for this diagnosis, and his dark urine is a likely result of increased uroporphyrin. As the acquired type of this disease typically presents in men with liver disease, he should be tested for HIV and HCV as well as fully evaluated for risk factors of alcoholic cirrhosis.

A 71-year-old man comes to the emergency department because of pain and swelling in his left leg that started after he cut his foot while swimming in the ocean. He has a history of alcoholic cirrhosis. His temperature is 38.3°C (101.0°F). Examination of the left foot shows a small, purulent wound with surrounding swelling and dusky redness extending to the midcalf. There are numerous hemorrhagic blisters and the entire lower leg is exquisitely tender to light palpation. There is no crepitus. Blood cultures grow gram-negative bacilli that ferment lactose. Which of the following is the most likely causal organism? This patient presents with typical signs of necrotizing fasciitis (e.g., severe pain, fever, dusky skin, blisters). His recent contact with seawater and history of cirrhosis suggests infection with a certain pathogen.

Vibrio vulnificus Vibrio vulnificus is a gram-negative, lactose-fermenting bacteria found in marine environments that can cause soft tissue infections, as well as gastroenteritis. Patients with underlying liver disease (e.g., cirrhosis, hemochromatosis) are at higher risk of developing severe infection (such as necrotizing fasciitis, as seen in this patient) and have higher mortality rates.

tially pink marks that progressed into blisters before ulcerating. He has also had a headache for 1 week. His temperature is 38.1°C (100.6°F). A photograph of one of the lesions is shown. There is pronounced edema of the surrounding skin and painless swelling of the left axillary lymph nodes. Which of the following is the greatest risk factor for this patient's condition? The picture shows a black eschar characteristic of cutaneous anthrax.

Wool handling Cutaneous anthrax is caused by Bacillus anthracis, which is most commonly transmitted via contact with infected animals or infected animal products (e.g., wool, hide, meat). The lesions in cutaneous anthrax typically begin as papules that expand and become painless eschars surrounded by extensive edema. Lymphadenopathy is also common. The disease can be accompanied by systemic symptoms such as fever, malaise, and headache and rarely progresses to bacteremia or death.

A 34-year-old woman with poorly controlled Crohn disease comes to the physician because of a 2-week history of hair loss and a rash on her face. She has also noticed that food has recently tasted bland to her. She had to undergo segmental small bowel resection several times because of intestinal obstruction and fistula formation. Examination shows several bullous, erythematous perioral plaques. There are two well-circumscribed circular patches of hair loss on the scalp. A deficiency of which of the following is the most likely cause of this patient's condition? Acrodermatitis enteropathica is a congenital cause of the same deficiency. It can also manifest with depressed immunity, resulting in frequent infections and, in males, decreased spermatogenesis.

Zinc Zinc deficiency can cause a constellation of clinical findings including immune dysfunction, impaired wound healing, hypogonadism, diarrhea, dermatitis, alopecia, abnormal sense of taste (dysgeusia), and abnormal sense of smell (dysosmia). Etiologies include poor dietary intake, impaired absorption (as a result of Crohn disease in this case), congenital conditions (e.g., acrodermatitis enteropathica), and liver or renal disease. Because zinc is mainly absorbed in the jejunum, this patient's Crohn disease and numerous small bowel resections likely substantially impede physiologic zinc absorption and predispose her to zinc deficiency.

A 16-year-old girl comes to the physician because of a mole on her face that she would like to get removed. The patient has recently moved to a new school and has been bullied because of her mole multiple times. She is also afraid she might "rip it off" when she washes her face or puts makeup on. She has no personal or family history of serious illness. She does not smoke cigarettes, drink alcohol, or use illicit drugs. She lives with her parents and grandmother. Vital signs are within normal limits. Physical examination shows a 5-mm brown papule above her right eyebrow, consistent with a melanocytic nevus. The physician explains that this type of skin lesion is generally benign and the risk of malignant transformation is low. The patient says that she is aware of the potential risks of the procedure and would like to proceed with the excision. Which of the following is the most appropriate response by the physician? When a minor requests nonemergent medical treatment, it is important to evaluate whether they are legally emancipated.

"I cannot perform this procedure unless your parents consent to it." This patient is a minor (< 18 years of age) who is legally unemancipated, so she requires parental consent for the surgical removal of her melanocytic nevus. Exceptions to the need for parental consent include emergent and lifesaving treatments (e.g., trauma, suicidal ideation), care regarding sex (e.g., contraception, sexually transmitted infections, pregnancy), and addiction care (e.g., drug addiction). In most cases, minors are considered legally emancipated if they live separately from their parents and are financially self-reliant, are married, are parents, are on active duty in the military, or have been declared legally emancipated by a court.

A 35-year-old man is brought to the emergency department 40 minutes after spilling hot oil over himself in a kitchen accident. Examination shows multiple tense blisters over the abdomen, anterior chest, and anterior and posterior aspects of the right upper extremity and right thigh. On deroofing the blisters, the skin underneath is tender, mottled, and sluggishly blanches with pressure. The skin over the left thigh is tender, erythematous, and shows quick capillary refill after blanching with pressure. Which of the following most closely approximates the body surface area affected by 2nd-degree burns in this patient? This patient has first and second degree burns. The body surface area affected by burns is approximately calculated using the rule of nines.

36% The rule of nines divides the body into 11 regions (head, anterior chest, abdomen, upper back, lower back, arms x2, thighs x2, lower legs x2), each of which is attributed a value of 9% body surface area. This patient has deep 2nd-degree burns, which are characterized by blister formation with nonblanching underlying skin, on his chest, abdomen, right arm, and right thigh, which corresponds to 36% of total body surface area. He also has a 1st-degree burn on the left thigh.

A 23-year-old man comes to the physician because of a painless swelling on the left side of his jaw for 2 months. It has been progressively increasing in size and is draining thick, foul-smelling fluid. He had a molar extracted 3 months ago. Examination shows a 4-cm, tender, erythematous mass in the left submandibular region with purulent drainage. There is submandibular lymphadenopathy. A culture of the purulent material shows catalase-negative, gram-positive filamentous rods that do not stain with carbol fuchsin. Which of the following is the most likely infectious pathogen? This organism forms yellow "sulfur" granules.

Actinomyces israelii Actinomyces israelii is a gram-positive, non-acid fast, rod-shaped anaerobe and part of normal oral flora. It causes cervical actinomycosis, which typically presents as a progressively enlarging mandibular nodule after oral surgery, as in this patient. The nodules can develop into purulent draining fistulae, which contain Actinomyces in characteristic yellow "sulfur" granules. The term "sulfur granules" reflects their yellow color, not their composition. Penicillin is the drug of choice for patients with actinomycosis. Surgical resection is indicated for patients who do not respond to antibiotic treatment.

A 15-year-old girl comes to the physician for a follow-up evaluation. She has multiple erythematous pustules and nodules over her face, for which she has received erythromycin and topical benzoyl peroxide. She is concerned that the therapy is ineffective. The physician recommends a drug on the condition that the patient agrees to use oral contraceptives. The molecular structure of the drug most likely recommended by the physician closely resembles a drug used to treat which of the following conditions? Patients with severe nodulocystic acne that is unresponsive to antibiotic therapy are most often prescribed isotretinoin, a member of the retinoid class of drugs.

Acute promyelocytic leukemia All-trans retinoic acid (ATRA) and isotretinoin are both retinoids. ATRA can be used in the treatment of acute promyelocytic leukemia, the M3 variant of acute myelogenous leukemia (AML). A t(15;17) translocation, which occurs in acute promyelocytic leukemia, changes the retinoic acid receptor so that granulocyte differentiation cannot occur under physiologic levels of retinoic acid. High doses of ATRA can induce remission by causing malignant cells to mature.

A 23-year-old woman comes to the physician because of a painless lesion on the arm that first appeared 5 days after she visited a petting zoo. The lesion began as a small papule and then became a blister that has since ulcerated. Her temperature is 38.3°C (101°F). Examination shows a black eschar with edematous borders on the left upper arm. There is axillary lymphadenopathy. The virulence factor responsible for the edema seen in this patient has a mechanism of action similar to which of the following enzymes? This woman has cutaneous anthrax, which typically manifests after a 5-7 day incubation period as a small papule that rapidly progresses to a painless, black eschar with extensive surrounding edema and regional lymphadenopathy. The edema is caused by edema factor produced by Bacillus anthracis.

Adenylate cyclase Edema factor (EF) produced by B. anthracis performs the same function as adenylate cyclase. After undergoing endocytosis, the EF causes a pathological increase in cAMP, which in turn drastically increases water influx into cells. This results in cellular lysis.

A 67-year-old man with hypertension comes to the physician because of a 5-month history of a facial rash. He occasionally feels burning or stinging over the affected area. His only medication is lisinopril. Physical examination shows the findings in the photograph. Which of the following is the strongest predisposing factor for this patient's skin condition? This patient presents with facial erythema, telangiectasia, papules, and pustules, which are findings characteristic of rosacea.

Alcohol consumption While the etiology of rosacea remains uncertain, there are several known triggers that precipitate or exacerbate symptoms, including stress, exercise, hot weather, and alcohol consumption. These triggers are believed to induce local inflammation, which causes vasodilation of cutaneous capillaries and skin flushing.

A 58-year-old woman comes to the physician because of an itchy rash on her leg 3 days after she returned from a camping trip with her grandchildren. Examination shows a linear, erythematous, maculopapular rash on the left lower extremity. Treatment with a drug is begun that is also effective for motion sickness. One hour later, she reports dry mouth. This adverse effect is most likely mediated through which of the following? The patient's allergic rash was most likely treated with a first-generation antihistamine (e.g., diphenhydramine). These agents are also used in the treatment of motion sickness.

Antagonism at acetylcholine receptors First-generation antihistamines, in addition to their H1-receptor antagonist action, also have potent acetylcholine receptor blocking action. AChRs are part of the parasympathetic nerve supply of the body. As saliva is secreted secondary to parasympathetic stimulation, antagonism at AChRs results in dry mouth, as seen in this patient. Other common anticholinergic effects include decreased sweating, tachycardia (unopposed sympathetic activity on the heart), blurring of vision for near objects (loss of accommodation reflex and mydriasis), urinary retention and constipation (loss of parasympathetic-mediated sphincter relaxation and peristalsis), and drowsiness.

A 12-year-old boy is brought to the emergency department by his mother because of progressive shortness of breath, difficulty speaking, and diffuse, colicky abdominal pain for the past 3 hours. Yesterday he underwent a tooth extraction. His father and a paternal uncle have a history of repeated hospitalizations for upper airway and orofacial swelling. The patient takes no medications. His blood pressure is 112/62 mm Hg. Examination shows edematous swelling of the lips, tongue, arms, and legs; there is no rash. Administration of a drug targeting which of the following mechanisms of action is most appropriate for this patient? This patient developed abdominal pain, swelling of the extremities (without urticaria or pruritus), and dyspnea secondary to edema of the lips and tongue after a triggering incident (dental work). The family history of recurrent orofacial swelling is indicative of hereditary angioedema, a disease caused by deficiency or dysfunction of C1 inhibitor.

Antagonist at bradykinin receptor The autosomal dominant defect in patients with hereditary angioedema causes a deficiency of C1 inhibitor, which physiologically suppresses steps in the complement pathways and inactivates kallikrein. Because kallikrein normally converts kininogen to bradykinin, increased levels of kallikrein lead to a buildup of bradykinin, and eventual angioedema. First-line therapies are focused either on antagonizing the bradykinin-B2-receptor (icatibant), preventing further generation of bradykinin with kallikrein inhibitor therapy (ecallantide), or replacing the defective enzyme with intravenous injection of purified C1-INH. Trauma, mental or physical stress, and dental work are common triggers for these acute attacks.

A 57-year-old woman comes to the physician because of a 1-month history of lesions on her eyelids. A photograph of the lesions is shown. This patient's eye condition is most likely associated with which of the following processes? The soft, yellow plaques on the eyelids are accumulations of lipid-laden macrophages (foam cells) known as xanthelasma.

Autoimmune destruction of lobular bile ducts Autoimmune destruction of lobular bile ducts and subsequent biliary obstruction is the underlying pathomechanism in primary biliary cholangitis. PBC is commonly associated with elevated serum cholesterol levels, as the major route of cholesterol disposal is through the synthesis and excretion of bile acids and bile salts. The cutaneous manifestations of hypercholesteremia are xanthelasma and xanthomas on the neck, trunk, shoulders, and axillae.

A 67-year-old man comes to the physician because of a lesion on his nose. The lesion appeared 1 year ago and has slowly increased in size since then. There is no associated pain or pruritus. He is otherwise healthy and has played golf every other day since his retirement. He says that he used to use tanning beds regularly in the past because he "liked the tanned look." A photograph of the lesion is shown. Which of the following is the most likely diagnosis? The photograph shows an ulcer with rolled borders and a central depression. A biopsy of the lesion would most likely show palisading nuclei.

Basal cell carcinoma Basal cell carcinoma (BCC), which is the most common type of malignant skin tumor, typically manifests as a slow-growing, nonhealing ulcer with a central depression and rolled borders (rodent ulcer), a pearly white nodule with superficial telangiectasis, or a scaling plaque (superficial BCC). This patient's history of using tanning beds and playing golf regularly increases his UV exposure, which is a risk factor for developing BCC. Facial BCC typically occurs above the line joining the earlobe and the corner of the mouth (e.g., on the forehead, eyelids, nose, nasolabial fold, upper lip). BCC is locally invasive but rarely metastasizes.

A 45-year-old homeless man comes to the emergency department because of a 1-week history of a red, itchy rash on his hands. He says the itching is worse at night and often wakes him from sleep. Several other residents of the homeless shelter where he is staying have reported similar symptoms. Physical examination shows the findings in the photograph. A topical drug with which of the following mechanisms of action is most likely to be effective? Pruritic, excoriated, erythematous papules with scattered vesicles on the interdigital folds and dorsum of the hand are a classic sign of scabies, which typically occurs in individuals living in crowded conditions (e.g., prisons, homeless shelters, nursing homes). Pruritis caused by scabies is often worse at night. Scabies is caused by the mite Sarcoptes scabiei and transmitted by direct human-to-human contact.

Binding to sodium channels Inhibition of voltage-gated sodium channels is the mechanism of action of permethrin, the first-line therapy for scabies. The blockade of sodium channels results in delayed repolarization of neuron cell membranes, which causes paralysis and death of the mite. Topical lindane solution is an alternative for the treatment of scabies. It inhibits GABA release and causes neurotoxicity and mite death.

A 47-year-old woman comes to the emergency department because of worsening pain, discoloration, and swelling of her right hand. Two days ago, she was camping outdoors in Texas with her two young children. She was awoken in the middle of the night by "something crawling on my hand." The following morning, she saw a wound on her hand that was initially painless, but over the next several hours she noticed reddening and blistering. Yesterday, the lesion turned blue with a blackish discoloration at the center. Her temperature is 38°C (100.4°F), pulse is 83/min, and blood pressure is 128/84 mm Hg. Physical examination shows a bluish plaque with a central area of necrosis surrounded by a rim of erythema at the dorsum of the right hand. Which of the following animals is most likely responsible for the patient's symptoms? This animal is most easily identified by the violin-shaped marking on its cephalothorax.

Brown recluse spider Brown recluse spiders are endemic to the southeastern and midwestern United States. They produce venom composed of various enzymes, some of which cause local destruction of tissue integrity, leading to the breakdown of skin, adipose tissue, vasculature, and eventual tissue death. Bites are initially painless but form into an erythematous, painful blister over the course of several hours. Progressive tissue death and bluish-black skin discoloration of the skin are seen in the first 24 hours, followed by tissue necrosis and eschar formation by the end of the first week. Systemic findings, although uncommon, include fever, malaise, nausea, and vomiting. Life-threatening complications are extremely rare.

A 20-year-old male comes to the physician because of a 1-week-history of a painless ulceration on his penis. The patient is sexually active with multiple partners and does not use barrier protection. Physical examination shows a shallow, firm ulcer with a smooth base along the shaft of the penis and nontender bilateral inguinal adenopathy. Which of the following is most likely to confirm the diagnosis in this patient? This patient's solitary, nontender, ulcerated lesion with indurated borders (chancre) located on the penis indicates early primary syphilis, which is caused by Treponema pallidum, a spirochetal organism.

Dark field microscopy Darkfield microscopy is a diagnostic method that allows direct visualization of T. pallidum from the lesion's serous exudate. This confirms the diagnosis without the use of a screening serologic test, which have a high false-negative rate in early primary syphilis.

A 3-year-old boy is brought to the physician for the evaluation of recurrent skin lesions. The lesions first appeared at the age of 3 months. He has also had several episodes of respiratory tract infections, enlarged lymph nodes, and recurrent fevers since birth. The boy attends daycare. His immunizations are up-to-date. He is at the 5th percentile for length and 10th percentile for weight. He appears ill. Temperature is 38°C (100.4°F). Examination shows several raised, erythematous lesions of different sizes over the face, neck, groin, and extremities; some are purulent. Bilateral cervical and axillary lymphadenopathy are present. Which of the following is the most likely underlying mechanism of this patient's symptoms? This patient presents with recurring respiratory infections, lymphadenopathy, and inflammatory, pustular lesions, starting at a very young age. Together, these features suggest impaired functioning of phagocytic cells, making him especially susceptible to infection with catalase-positive organisms (e.g., S. aureus, E. coli, Aspergillus).

Defective NADPH oxidase The NADPH oxidase enzyme is responsible for the respiratory burst (oxidative burst) within phagolysosomes, a component of phagocytic cells, which causes a rapid release of reactive oxygen species (ROS) that can destroy bacteria. In patients with defective NADPH oxidase, also known as chronic granulomatous disease (CGD), this reaction does not occur. This leads to impaired function of phagocytic cells and makes patients susceptible to infection, as in this patient, usually with catalase-positive organisms. The nitroblue tetrazolium test and the dihydrorhodamine flow cytometry test are used to check for NADPH oxidase activity and to diagnose chronic granulomatous disease.

A 62-year-old woman comes to the physician because of a 2-month history of a rash on her ankles with intermittent itching. After the rash developed, she started applying a new scented lotion to her legs daily. She works as a cashier at a grocery store. She has type 2 diabetes mellitus and hypertension. Current medications include metformin and enalapril. Examination shows enlarged superficial veins of the right lower extremity and red-brown discoloration with indistinct margins over the medial ankles. There is 1+ edema in the lower extremities. Which of the following is the most likely cause of this patient's skin findings? This woman has a pruritic, red-brown rash over the medial ankles, varicose veins, and bilateral lower extremity edema. Her occupation likely requires her to stand for prolonged periods. These findings suggest stasis dermatitis due to chronic venous disease.

Dermal deposition of hemosiderin Dermal deposition of hemosiderin is a key feature of stasis dermatitis. Elevated venous pressures from chronic venous disease result in red blood cell breakdown, which leads to hemosiderin production. Hemosiderin penetrates defective capillaries and accumulates in the dermis, which causes a pruritic, red-brown rash, as seen in this patient. The medial ankles are the most frequently involved site.

An investigator is studying DNA repair processes in an experimental animal. The investigator inactivates a gene encoding a protein that physiologically excises nucleotides from damaged, bulky, helix-distorting DNA strands. A patient with a similar defect in this gene is most likely to present with which of the following findings? Pyrimidine dimers cause bulky, helix-distorting DNA lesions that are normally repaired by excision endonucleases. A defect in any one of these enzymes causes xeroderma pigmentosum.

Dry skin and increased photosensitivity Dry skin and increased photosensitivity are characteristic features of xeroderma pigmentosum. UV radiation (both UVA and UVB) leads to the formation of pyrimidine dimers (usually thymine), which are unable to be repaired by defective excision endonucleases. Even with aggressive avoidance of sun exposure, the accumulation of thymine dimers eventually causes malignant skin cancer, beginning in early childhood.

A 54-year-old woman comes to the physician because of a 6-month history of dull, persistent pain and swelling of her right leg. The pain is worse at the end of the day and is relieved by walking or elevating her feet. Two years ago, she developed acute deep vein thrombosis in her right calf after a long flight, which was treated with anticoagulants for 6 months. Physical examination shows 2+ pitting edema of her right leg. The skin around the right ankle shows a reddish-brown discoloration and multiple telangiectasias. She has dilated varicose veins in the right leg. Which of the following is most likely to establish the diagnosis? This patient's unilateral chronic dull pain, edema, dilated veins in the lower extremity, and history of DVT suggest chronic venous insufficiency.

Duplex ultrasonography Duplex ultrasonography (compression ultrasonography) is the first-line investigation in patients with features suggestive of chronic venous insufficiency (CVI), as seen here. Evidence of venous reflux on duplex ultrasonography confirms the diagnosis of CVI. Duplex ultrasonography can help determine the course of treatment by identifying which valves are incompetent and assessing the patency of the previously thrombosed deep veins.

A 23-year-old woman comes to the physician because of a 5-month history of a pruritic rash on the cubital fossae of the upper extremities. She has no history of serious illness and takes no medications. A skin biopsy of the rash shows intraepidermal accumulation of edematous fluid and widening of intercellular spaces between keratinocytes. Which of the following is the most likely diagnosis? Fluid collections within the epidermis seen on histopathological examination indicate spongiosis.

Eczematous dermatitis Eczematous dermatitis is the umbrella term for multiple conditions with overlapping clinical and histopathological features, including atopic dermatitis and allergic contact dermatitis. Each type manifests as a pruritic, erythematous, maculopapular rash with occasional crusting, drainage of serous fluid, and vesicle or bulla formation. This patient's 5-month history of a rash on both arms suggests a diagnosis of atopic dermatitis, a chronic condition that typically develops on flexural surfaces (e.g., the antecubital fossae) in childhood but that can also manifest in adulthood.

An 11-year-old girl is brought to the physician by her parents because of a mildly pruritic rash on her trunk and extremities for 2 days. One week ago, she developed a low-grade fever, rhinorrhea, and headache, followed by a facial rash 4 days later. The facial rash did not involve the perioral skin. Her temperature is 37.4°C (99.3°F). A photograph of the rash on her lower arms is shown. Which of the following is the most likely diagnosis? The causal pathogen of this condition can cause hydrops fetalis in case of intrauterine transmission during pregnancy and aplastic crises in patients with sickle cell disease.

Erythema infectiosum The patient's presentation of prodromal low-grade fever and cold‑like symptoms, followed by a facial maculopapular rash with perioral sparing ("slapped-cheek" appearance) that spreads to the trunk and extremities with a lace-like pattern, is consistent with erythema infectiosum, also known as fifth disease. Fifth disease is caused by a parvovirus B19 infection. It is most commonly seen in children aged 5-15 years and is self-limiting, usually resolving in 7-10 days. Parvovirus B19 infection in pregnancy can cause severe anemia, fetal hydrops, and fetal demise.

A 32-year-old woman with a recurrent vesicular genital rash comes to the physician because of a 3-day history of a painful, pruritic rash that began on the extremities and has spread to her trunk. Her only medication is acyclovir. Her temperature is 38.1°C (100.6°F). Examination of the skin shows several reddish-purple papules and macules, some of which have a dusky center with a lighter ring around them. Which of the following is the most likely diagnosis? This patient's rash is most commonly associated with herpes simplex infections and is characterized by target lesions (macules with a dusky center surrounded by a lighter ring).

Erythema multiforme This patient with recurrent genital HSV and a painful exanthem with target lesions most likely has Erythema multiforme (EM). EM is a type IV hypersensitivity reaction of the skin that can be triggered by certain infections (e.g., HSV, Mycoplasma pneumonia) and medications (e.g., beta-lactam antibiotics, sulfonamides, phenytoin). EM is characterized by lesions of varying morphology (e.g., macules, papules, vesicles) that typically progress to target lesions and spread proximally from the backs of the hands and feet, as seen here. Nikolsky sign is negative and the disease course is self-limiting in most cases.

A 62-year-old woman with metastatic breast cancer comes to the physician because of a 2-day history of fever, chills, and new gluteal lesions. The lesions began as painless red macules and evolved into painful ulcers overnight. She received her fourth course of palliative chemotherapy 2 weeks ago. Her temperature is 38.2°C (100.8°F). Laboratory studies show a leukocyte count of 2,000/mm3 (20% segmented neutrophils). A photograph of one of the skin lesions is shown. Which of the following virulence factors is most likely involved in the pathogenesis of this patient's skin finding? This patient's chemotherapy has caused immunosuppression and neutropenia, which increase the risk of developing an infection. She has developed a fever and rapidly evolving gluteal lesions characterized by a circular ulcer with surrounding erythema and a central black eschar. These features are consistent with ecthyma gangrenosum, which is classically associated with P. aeruginosa bacteremia.

Exotoxin A Exotoxin A is an AB toxin produced by P. aeruginosa that inhibits protein synthesis by inactivating elongation factor (EF-2). This leads to cell death, which is responsible for the lesions of ecthyma gangrenosum. Other virulence factors produced by P. aeruginosa that also cause cell death include phospholipase C (which degrades cell membranes), elastase (which causes proteolytic damage to tissue, destroying blood vessel walls), and pyocyanin (which creates damaging free radicals and contributes to the blue-green color of some infections). P. aeruginosa also produces endotoxin, which causes fever and shock.

tendinous xanthomaA previously healthy 22-year-old man comes to the physician because of multiple nodules on his hands that first appeared a few months ago. He works as a computer game programmer. His father died of a myocardial infarction at 37 years of age, and his mother has rheumatoid arthritis. A photograph of the lesions is shown. The nodules are firm, mobile, and nontender. Which of the following is the most likely mechanism underlying this patient's skin findings? The firm, skin-colored nodules on the patient's hands are tendinous xanthomas.

Extravasation of lipoproteins Tendinous xanthomas form from plasma lipoproteins that have extravasated and accumulated in tendons (most commonly the extensor tendons of the hands and the Achilles tendon) in patients with extremely high levels of circulating lipids. These findings and the death of this patient's father from a myocardial infarction at 37 years of age should raise suspicion for familial hypercholesterolemia, which increases the risk of developing premature coronary artery disease.

A 62-year-old woman comes to the physician for evaluation of a mole on her forearm that has increased in size over the last several months. Physical examination shows a 9-mm skin lesion on the right forearm with irregular borders. An excisional biopsy is performed, and genetic analysis shows a mutation in the gene that encodes B-Raf. Which of the following cellular events most likely predisposed this patient to developing this skin lesion? This woman's history of an evolving skin lesion with irregular borders in a sun-exposed area is suspicious for malignant melanoma, a cancer that often has an underlying V600E mutation in the BRAF gene (which encodes B-Raf kinase). An important risk factor for developing malignant melanoma is UV radiation exposure, which damages DNA.

Formation of covalent bonds between adjacent pyrimidine bases The formation of covalent bonds between adjacent thymine and cytosine bases creates pyrimidine dimers. These DNA lesions are caused by exposure to UV radiation and form bulky helix distortions that interfere with DNA replication. This increases the risk of developing gain-of-function mutations in the BRAF gene. These mutations result in overactivation of growth signals that can lead to malignant melanoma.

A 24-year-old man comes to the physician for the evaluation of a severely pruritic skin rash. Physical examination shows a symmetrical rash over the knees and elbows with tense, grouped vesicles, and several excoriation marks. Microabscesses in the papillary dermis are seen on light microscopy. Immunofluorescence shows deposits of immunoglobulin A at the tips of dermal papillae. This patient's skin findings are most likely associated with which of the following? This patient's skin findings (symmetrical, pruritic, grouped, subepidermal vesicles on extensor surfaces) are manifestations of dermatitis herpetiformis. The severe pruritus can lead to excoriations, as seen here. Papillary deposits of IgA confirm the diagnosis.

Gliadin-dependent hypersensitivity Gliadin-dependent hypersensitivity causes celiac disease, a condition strongly associated with dermatitis herpetiformis (DH). The majority of patients with DH have gastrointestinal disease related to gluten sensitivity on histopathologic examination, despite lacking gastrointestinal symptoms (e.g., bloating, diarrhea, abdominal pain). In addition to lifelong avoidance of gluten-containing foods, the first-line treatment of DH is dapsone.

A previously healthy 25-year-old man comes to the physician because of a painful lesion on his penis. The lesion appeared 2 days ago and has since grown in size. It is draining yellow purulent material. One week ago, he returned from a 1-month trip to Cambodia. His immunizations are up-to-date. He is sexually active with multiple female partners and uses condoms inconsistently. His temperature is 37.2°C (99.0°F). Physical examination shows tender inguinal lymphadenopathy. A photograph of the lesion is shown. Which of the following is the most likely causal organism? This patient's painful and necrotic lesion with purulent discharge as well as tender lymphadenopathy following recent travel to a tropical region (i.e., Cambodia) is suggestive of chancroid.

Haemophilus ducreyi Infection with Haemophilus ducreyi causes chancroid, which typically manifests with an intensely painful, clearly demarcated lesion with a grey necrotic base and painful inguinal lymphadenopathy, as seen in this patient. Men usually present with a single lesion, while women often develop multiple synchronous lesions. H. ducreyi is endemic in tropical and subtropical regions (e.g., Cambodia) and rare in the United States. Chancroid is usually treated with a single dose of oral azithromycin or intramuscular ceftriaxone.

A 5-year-old boy is brought to the physician by his parents because of a 4-day history of arthralgia, abdominal pain, and lesions on his arms and legs. Ten days ago, he had an upper respiratory tract infection. A photograph of his right leg is shown. Further evaluation is most likely to show which of the following? The onset of abdominal pain, arthralgia, and purpura several days after an infection in a child < 10 years of age should raise suspicion of IgA vasculitis, also known as Henoch-Schonlein purpura.

Hematuria IgA vasculitis is characterized by the following tetrad of clinical findings: palpable purpura, arthralgias, GI symptoms such as abdominal pain, and renal disease, most commonly manifesting as nephritic syndrome. It is hypothesized that the deposition of IgA immune complexes in vascular walls activates the complement system, thereby leading to vascular inflammation and damage. Involvement of the renal capillaries and mesangium frequently leads to hematuria, often with RBC casts.

A 31-year-old man comes to the physician because of a 2-day history of nausea, abdominal discomfort, and yellow discoloration of the eyes. Six weeks ago, he had an episode of fever, joint pain, swollen lymph nodes, and an itchy rash on his trunk and extremities that persisted for 1 to 2 days. He returned from a backpacking trip to Colombia two months ago. His temperature is 39°C (101.8°F). Physical examination shows scleral icterus. Infection with which of the following agents is the most likely cause of this patient's findings? This patient's current condition (jaundice, nausea) suggest liver pathology and his medical history suggests a serum sickness-like reaction (fever, arthritis, itchy rash).

Hepatitis B Hepatitis B is associated with a serum sickness-like reaction, which typically occurs during the prodromal period of the infection 1-2 weeks after antigen exposure and is thought to be mediated by immune complex formation. Symptoms of serum sickness-like reaction usually resolve within a few weeks, but symptoms from acute HBV infection may persist for longer, as seen in this patient. Other causes of serum sickness-like reaction include antibiotics (cefaclor, penicillin), streptococcal infection, and vaccines.

A 3-year-old boy is brought to the physician for evaluation of a generalized, pruritic rash. The rash began during infancy and did not resolve despite initiating treatment with topical corticosteroids. Three months ago, he was treated for several asymptomatic soft tissue abscesses on his legs. He has been admitted to the hospital three times during the past two years for pneumonia. Physical examination shows a prominent forehead and a wide nasal bridge. Examination of the skin shows a diffuse eczematous rash and white plaques on the face, scalp, and shoulders. Laboratory studies show a leukocyte count of 6,000/mm3 with 25% eosinophils and a serum IgE concentration of 2,300 IU/mL (N = 0-380). Flow cytometry shows a deficiency of T helper 17 cells. The patient's increased susceptibility to infection is most likely due to which of the following? In combination with his increased serum IgE, eosinophilia, and deficiency of Th17 cells, this patient's coarse facies (prominent forehead, wide nasal bridge), history of eczema, cold abscesses, and recurrent pneumonia are consistent with autosomal dominant hyperimmunoglobulin E syndrome (Job syndrome).

Impaired chemotaxis of neutrophils Autosomal dominant hyperimmunoglobulin E syndrome (hyper-IgE syndrome) is caused by a mutation in the STAT3 gene, which leads to Th17 cell deficiency and decreased Th17-dependent production of IL-17. This results in impaired neutrophil chemotaxis, which causes decreased inflammation, cold abscesses (most commonly due to Staphylococcus aureus), and increased susceptibility to sinopulmonary infections (e.g., pneumonia). In addition to this patient's findings, hyper-IgE syndrome is also associated with other skeletal abnormalities (e.g., retained baby teeth, osteopenia/osteoporosis, pathologic fractures), and decreased IFN-γ.

A 42-year-old woman with hypertension comes to the physician because of a 2-month history of persistent reddening of her face, daytime fatigue, and difficulty concentrating. She has fallen asleep multiple times during important meetings. Her only medication is lisinopril. She is 170 cm (5 ft 7 in) tall and weighs 88 kg (194 lb); BMI is 30 kg/m2. Her blood pressure is 145/85 mm Hg. Physical examination shows erythema of the face that is especially pronounced around the cheeks, nose, and ears. Fasting serum glucose concentration is 96 mg/dL. Which of the following is the most likely cause of this patient's facial discoloration? This overweight patient has increased daytime sleepiness and impaired concentration, making obstructive sleep apnea (OSA) a likely diagnosis.

Increased erythropoietin production In obstructive sleep apnea (OSA), nighttime obstruction of the upper airways (e.g., resulting from obesity) leads to apneic episodes, hypoxia, and poor quality of sleep. Sleep deprivation causes daytime sleepiness and impaired concentration, while hypoxia triggers renal erythropoietin secretion and increased production of RBCs in the bone marrow, resulting in a characteristic red discoloration of the face (plethora). Chronic hypoxia can also trigger a sympathetic response, which could explain this patient's hypertension.

A 23-year-old woman comes to the physician for evaluation of two masses on her right auricle for several months. The masses appeared a few weeks after she had her ear pierced and have increased in size since then. A photograph of her right ear is shown. Which of the following is the most likely cause of these findings? This patient's history of trauma, followed by tissue growth beyond the boundaries of the original lesion is suggestive of keloid formation. The brownish-red color of the lesion shown supports this diagnosis.

Increased production of hyalinized collagen Keloids are formed from the hyperproliferation of fibroblasts, which results in disorganized overproduction of hyalinized collagen types I and III. Keloids are most commonly seen in dark-skinned individuals. Common locations for keloids include the earlobes, face (e.g., cheeks), upper extremities, chest, and neck.

A 72-year-old woman with type 2 diabetes mellitus comes to the physician because she is concerned about the appearance of her toenails. Examination shows yellowish discoloration of all toenails on both feet. The edges of the toenails are lifted, and there is subungual debris. Potassium hydroxide preparation of scrapings from the nails shows multiple branching septate hyphae. Treatment with oral terbinafine is begun. Which of the following is the primary mechanism of action of this drug? Yellow discoloration, onycholysis (separation of the nail plate and the nail bed), and the formation of subungual (keratinous) debris under the toenails of this elderly patient with diabetes suggest a diagnosis of onychomycosis, which is most commonly caused by dermatophyte infection. Visualization of branching septate hyphae on potassium hydroxide (KOH) preparation confirms the diagnosis. Oral terbinafine therapy is the treatment of choice for onychomycosis.

Inhibition of squalene epoxidase Terbinafine inhibits squalene epoxidase, a key enzyme in fungal sterol biosynthesis that converts squalene to squalene epoxide. The resulting deficiency of ergosterol within the fungal cell membrane leads to cell death.

A 57-year-old man comes to the physician because of tiredness and dyspnea on exertion for several months. Recently, he has also noticed changes of his fingernails. A photograph of his nails is shown. Which of the following is the most likely underlying cause of these findings? This image shows a spoon-shaped or concave abnormality of the nail plate known as koilonychia.

Iron deficiency anemia Koilonychia is a finger nail change observed in iron deficiency anemia, which also presents with tiredness and dyspnea as seen in this patient. These changes are thought to be caused by weakening of nail connective tissue due to poor blood flow and reduced activity of iron‐containing enzymes. In resource-rich settings, the most common cause of iron deficiency anemia in adult men is gastrointestinal bleeding, which is why after confirming this patient's iron deficiency, additional evaluations for possible causes should be performed.

A 3-year-old boy is brought to the emergency department after losing consciousness. His parents report that he collapsed and then had repetitive, twitching movements of the right side of his body that lasted approximately one minute. He recently started to walk with support. He speaks in bisyllables and has a vocabulary of almost 50 words. Examination shows a large purple-colored patch over the left cheek. One week later, he dies. Which of the following is the most likely finding on autopsy of the brain? This patient's condition is associated with somatic mutation of the GNAQ gene.

Leptomeningeal vascular malformation This patient has several characteristics of Sturge-Weber syndrome, a congenital disorder that affects the capillaries and commonly manifests with developmental delay and port-wine stain. Sturge-Weber syndrome is associated with vascular malformations in the choroid and CNS, including leptomeningeal angiomas, which are typically located ipsilateral to the port-wine stain and can cause seizures.

One week after starting a new medication, a 16-year-old girl is brought to the emergency department by her mother because of a painful, blistering rash. She has a history of bipolar disorder. Her temperature is 39°C (102°F). Physical examination shows numerous coalescing bullae with epidermal detachment covering the face, trunk, and extremities. There are hemorrhagic erosions on the hard palate and buccal mucosa. When lateral pressure is applied to healthy-appearing skin at the edge of a bulla, a blister starts to form. Which of the following drugs is most likely responsible for this patient's current condition? This girl's fever, blistering rash, positive Nikolsky sign, and recent medication change are consistent with a diagnosis of Stevens-Johnson syndrome (SJS). Toxic epidermal necrolysis is a severe form of this condition that involves > 30% of the skin.

Lamotrigine Lamotrigine is an antiepileptic drug with mood-stabilizing properties that is used to treat bipolar disorder. It is one of the most commonly implicated triggers of SJS. Most cases of SJS develop within 2-8 weeks of initiating the offending medication. The risk of developing SJS induced by lamotrigine therapy is greatest when high doses are used, so the initial dose should be low and increased slowly. In addition, about 10% of patients treated with lamotrigine develop a benign skin rash that is difficult to differentiate from early SJS; accordingly, all patients should be advised to discontinue lamotrigine therapy at the first sign of a rash.

A 34-year-old man comes to the physician for a 2-month history of an itchy rash on his forearm. He feels well otherwise and has not had any fever or chills. He returned from an archaeological expedition to Guatemala 4 months ago. Skin examination shows a solitary, round, pink-colored plaque with central ulceration on the right wrist. There is right axillary lymphadenopathy. A photomicrograph of a biopsy specimen from the lesion is shown. Which of the following is the most likely causal organism? Contains overlay The photograph shows macrophages with cytoplasmic amastigotes. The causal organism is transmitted by phlebotomine sand flies.

Leishmania braziliensis Leishmania braziliensis is the most likely causal organism in a patient with a history of recent travel to Guatemala who presents with a chronic skin ulcer and has a biopsy showing macrophages with amastigotes. L. braziliensis is transmitted to humans via infected sandflies and usually begins as a reddish papule around the sandfly bite that then enlarges into a nodule, oftentimes with central ulceration. Definitive diagnosis of cutaneous leishmaniasis requires detection of the pathogen in a skin biopsy, as seen here.

An 8-year-old girl is brought to the emergency department because of a 2-day history of an intermittent, diffuse abdominal pain. She has also had a nonpruritic rash on her legs and swelling of her ankles for 1 week. Two weeks ago, she had a sore throat, which was treated with oral amoxicillin. Examination of the lower extremities shows non-blanching, raised erythematous papules. The ankle joints are swollen and warm, and their range of motion is limited by pain. Laboratory studies show a platelet count of 450,000/mm3. Test of the stool for occult blood is positive. Which of the following is the most likely diagnosis? This child's abdominal pain, arthritis, and raised rash are indicative of immune complex deposition disease.

Leukocytoclastic vasculitis Leukocytoclastic vasculitis is an umbrella term for vasculitides, including Henoch-Schönlein purpura (HSP). HSP is an acute immune-complex mediated vasculitis of the small vessels that is often preceded by an upper respiratory infection. The patient's history of pharyngitis 2 weeks ago and the palpable non-blanching skin lesions with arthritis and abdominal pain are classic signs of HSP. HSP can also present with bloody stools or occult bleeding (as in this patient) but is accompanied by a raised platelet count, which helps to differentiate it from thrombocytopenia leading to intestinal bleeding and petechiae.

A 47-year-old woman comes to the physician because of a 1-month history of progressive weakness. She has had increased difficulty climbing stairs and standing from a seated position. She takes no medications. Neurologic examination shows weakness of the proximal muscles. Skin examination shows diffuse erythema of the upper back, posterior neck, and shoulders. A photograph of the patient's eye is shown. Antibodies against which of the following are most likely to be present in this patient? This patient has dermatomyositis, which typically manifests with proximal muscle weakness, a heliotrope rash, and the shawl sign.

Mi-2 protein Anti-Mi-2 antibodies are associated with dermatomyositis and polymyositis. These antibodies, along with anti-Jo-1 antibodies and anti-SRP antibodies, are relatively specific for these conditions. Dermatomyositis is associated with several malignancies (e.g., ovarian adenocarcinoma), and all patients diagnosed with this condition should be evaluated with age-appropriate screening.

A 30-year-old man comes to the physician for a follow-up examination 1 month after sustaining a chemical burn over the dorsum of his right hand and forearm. Physical examination shows hyperextension of the hand at the wrist. The skin over the dorsum of the wrist is tense and there is a thick, epithelialized scar. Range of motion of the right wrist is restricted. This patient's contracture is most likely due to activity of which of the following cells? Wound contraction occurs during the proliferative phase of wound healing. This process is not only facilitated by a special cell type but is also mediated by metalloproteinase secretion.

Myofibroblasts Myofibroblasts, derived from fibroblasts, generate a contractile force that draws wound edges together. They are present in the proliferative stage of wound healing (days 3-7) and are lost through apoptosis when wound healing is complete. However, excessive proliferation and persistence of myofibroblasts lead to pathologic fibrosis and contracture, which causes functional impairment, as is the case here.

A 42-year-old woman with well-controlled HIV on antiretroviral therapy comes to the physician because of a 2-week history of a painless lesion on her right calf. Many years ago, she had a maculopapular rash over her trunk, palms, and soles that resolved spontaneously. Physical examination shows a 4-cm firm, nontender, indurated ulcer with a moist, dark base and rolled edges. There is a similar lesion at the anus. Results of rapid plasma reagin testing are positive. Which of the following findings is most likely on microscopic examination of these lesions? This patient's current skin lesions are most likely gummas, a sign of tertiary syphilis. Her remote history of rash is suggestive of secondary syphilis.

Necrotic area surrounded by fibroblast and macrophage infiltrate Gummas are granulomatous skin lesions characteristic of tertiary syphilis. A biopsy of a gumma would most likely show a central area of necrosis (due to obliterating endarteritis, which causes ischemia of the lesion's center) surrounded by an inflammatory cellular infiltrate (due to a type 4 hypersensitivity reaction to Treponemal spirochetes). Gummas may form on the skin, CNS (neurosyphilis), bones, and/or internal organs. They appear in advanced Treponemal pallidum infections and are more common in patients with HIV infection.

A 16-year-old boy is brought to the physician by his host parents for evaluation of a progressively pruritic rash over his shoulders and buttocks for the past 6 months. He recently came to the United States from Nigeria to attend a year of high school. He reports that it has been increasingly difficult for him to read the whiteboard during classes. Physical examination shows symmetrically distributed papules 4-8 mm in diameter, excoriation marks, and patchy hyperpigmentation over his shoulders, waist, and buttocks. There is nontender inguinal lymphadenopathy and several firm, nontender subcutaneous nodules along the right iliac crest. Six skin snip biopsies are taken from the pelvic girdle, buttocks, and thigh, and are then incubated in saline. After 24 hours, microscopic examination shows motile microfilariae. Which of the following is the most likely diagnosis? This patient presents with a parasitic disease that is endemic to Nigeria and transmitted by the female blackfly.

Onchocerciasis Onchocerciasis (river blindness) is caused by the filarial nematode Onchocerca volvulus and is transmitted via the bite of the blackfly (Simulium spp.), the intermediate host that inoculates O. volvulus larvae into human skin. The larvae mature into adult filaria and become encapsulated within subcutaneous nodules (onchocercomas), which are typically located over bony prominences (e.g., iliac crest). The filaria produce thousands of microfilaria, which migrate through the skin, lymphatics, and blood to reach the lymph nodes, eyes, and deep organs. The inflammatory response to these microfilariae can cause ocular disease (e.g., keratitis, uveitis) as well as an intensely pruritic papular dermatitis with secondary skin changes due to itching (e.g., excoriations, hyperpigmentation). Onchocerciasis is diagnosed by the presence of microfilaria in skin snip biopsies and treated with ivermectin.

A 37-year-old woman comes to the physician because of a 6-month history of weight loss, bloating, and diarrhea. She does not smoke or drink alcohol. Her vital signs are within normal limits. She is 173 cm (5 ft 8 in) tall and weighs 54 kg (120 lb); BMI is 18 kg/m2. Physical examination shows bilateral white spots on the temporal half of the conjunctiva, dry skin, and a hard neck mass in the anterior midline that does not move with swallowing. Urinalysis after a D-xylose meal shows a normal increase in renal D-xylose excretion. Which of the following is most likely to have prevented this patient's weight loss? This patient has xeroderma and keratin deposits on the conjunctivae (Bitot's spots), which are features suggestive of vitamin A deficiency. The physiologic increase in D-xylose excretion seen in this patient indicates that the integrity of the intestinal mucosa is intact.

Pancreatic enzyme replacement This patient's hard, immobile neck mass in the anterior midline is suggestive of Riedel's thyroiditis, which is associated with other IgG4-related systemic diseases. Her xeroderma and Bitot's spots indicate a vitamin A deficiency, and her weight loss, diarrhea, and physiologic increase in urinary D-xylose excretion indicate exocrine pancreatic insufficiency, most likely due to autoimmune pancreatitis. Pancreatic enzyme replacement would be the preventative treatment.

A 52-year-old woman with type 2 diabetes mellitus comes to the physician because of a 2-day history of blisters on her forearms and pain during sexual intercourse. Her only medications are metformin and glyburide. Examination reveals multiple, flaccid blisters on the volar surface of the forearms and ulcers on the buccal, gingival, and vulvar mucosa. The epidermis on the forearm separates when the skin is lightly stroked. Which of the following is the most likely diagnosis? This patient's condition is confirmed via biopsy, which would show acantholysis and deposition of IgG in the intercellular spaces of the epidermis.

Pemphigus vulgaris Pemphigus vulgaris is an autoimmune blistering disorder caused by autoantibodies against desmosomal adhesion proteins, including desmoglein 3 and desmoglein 1. Certain medications, such as captopril, glyburide, thiol-containing drugs (e.g., penicillamine), and phenol-containing drugs (e.g., cephalosporins, rifampicin, phenobarbital, aspirin), can induce the formation of these autoantibodies. Since desmosomal adhesion proteins enable keratinocyte adherence, damage to these proteins causes intraepidermal bullae formation and a positive Nikolsky sign. The lesions typically occur first on mucous membranes and can become confluent or erode to form painful ulcers.

A 17-year-old girl comes to the physician because of a 1-week history of severe itching in the area of her genitals. She reports that the itching is most severe at night. She has been sexually active with three partners over the past year; she uses condoms for contraception. Her current sexual partner is experiencing similar symptoms. Pelvic examination shows vulvar excoriations. A photomicrograph of an epilated pubic hair is shown. Which of the following is the most likely causal organism? The photomicrograph shows a nit attached to the hair shaft.

Phthirus pubis Infestation with the parasite Phthirus pubis causes pediculosis pubis, an infection that manifests with anogenital pruritus. Pediculosis pubis is a sexually transmitted infection that is not prevented by condoms. Physical examination of the pubic area typically shows excoriations as well as lice, their nits, and their droppings, which are visible both with the naked eye and on microscopy, as seen in the photomicrograph here.

A 31-year-old woman comes to the physician because of new-onset skin lesions on her face. She is newlywed and got back from her honeymoon in South Africa 4 months ago. She reports that the lesions have increased in size since the beginning of the summer. She does not smoke or drink alcohol. She takes no medications. Physical examination shows skin lesions scattered across her forehead and malar region, with smaller lesions on the nose and above the upper lip. A photograph of her forehead is shown. There are no further lesions on the remainder of her skin. This patient's skin findings are most likely associated with which of the following? The hyperpigmented macules seen in this patient are consistent with melasma.

Pregnancy Melasma (chloasma) is a benign disorder characterized by irregular, patchy hyperpigmentation of the skin that is most commonly seen in women of reproductive age. Hormonal changes (e.g., pregnancy, oral contraceptive use) seem to play an important role in the development of this condition. Melasma can be triggered and/or exacerbated by exposure to sunlight. Avoiding sunlight and/or applying sunscreen during pregnancy can prevent or slow the progression of melasma. Topical treatment with depigmenting agents such as hydroxyquinone may be used in severe cases. Linea nigra and areolar darkening can also develop during pregnancy as a result of endocrine changes but, unlike melasma, these two forms of cutaneous hyperpigmentation are not associated with sun exposure.

A 59-year-old man with chronic hepatitis C infection comes to the physician because of a 2-week history of ankle pain and nonpruritic skin lesions on his legs. He does not recall recent trauma or injury. He has not received treatment for hepatitis. Examination shows diffuse, violaceous lesions on both lower extremities. The lesions are 4-7 mm in size, slightly raised, and do not blanch with pressure. These skin lesions are best classified as which of the following? Patients with chronic hepatitis C infection can develop cryoglobulinemia, which is characterized by deposition of immune complexes and subsequent vasculitis.

Purpura Palpable purpura are raised, violaceous lesions caused by bleeding from small vessels in the skin. Because blood leaks into the extracellular space, these lesions do not completely blanch with pressure (in contrast to telangiectasias). This patient's skin lesions and arthritis are likely caused by cryoglobulinemic vasculitis, in which cryoglobulin immune complexes are deposited in small vessel walls, causing inflammation.

A 68-year-old man comes to the emergency department 12 hours after the appearance of tender, purple discolorations on his thighs and lower abdomen. He began taking a medication 4 days ago after failed cardioversion for atrial fibrillation, but he cannot remember the name. Physical examination shows a tender bluish-black discoloration on the anterior abdominal wall. A photograph of the right thigh is shown. Which of the following is the most likely explanation for this patient's skin findings? The patient was likely started on warfarin and now has warfarin-induced skin necrosis.

Reduced levels of protein C Reduced levels of protein C can be caused by warfarin. When warfarin therapy is initiated, the levels of anticoagulative protein C and protein S are diminished before the vitamin K-dependent factors II, VII, IX, X. This results in a transient state of hypercoagulability, during which time patients are at risk of thrombosis and warfarin-induced skin necrosis.

A 36-year-old woman comes to the emergency department because of an itchy lesion on her skin. The rash developed shortly after she took an antibiotic for a urinary tract infection. Her temperature is 37.5°C (99.3°F), pulse is 99/min, and blood pressure is 100/66 mm Hg. Physical examination shows swelling of the face and raised, erythematous plaques on her trunk and extremities. Which of the following is the most likely cause of this patient's current condition? The patient's acute-onset swelling and urticaria shortly after ingestion of an antibiotic are characteristic of a type I hypersensitivity reaction.

Release of a vasoactive amine The release of histamine, a vasoactive amine, plays an integral role in the type I hypersensitivity reaction seen in this patient. IgE antibodies from a previous encounter with a certain antigen are bound to mast cells and basophils. Re-exposure leads to degranulation of IgE-coated mast cells with release of histamine and various other mediators responsible for the characteristic features of type I HSR (i.e., erythema, pruritus, and urticaria). The patient's hypotension is due to hypovolemia caused by peripheral vasodilation and increased vascular permeability.

A 73-year-old woman is brought to the physician by her son because of a 2-month history of diarrhea and recurrent upper respiratory tract infections. Her son says that she frequently trips over the nightstand when she gets up to go to the bathroom at night. She is 173 cm (5 ft 8 in) tall and weighs 54 kg (120 lb); BMI is 18 kg/m2. Physical examination shows dry skin, multiple bruises on the shins, and triangular keratin plaques on the temporal half of the conjunctiva. A deficiency of which of the following is the most likely underlying cause of these findings? This malnourished patient presents with xerosis cutis and Bitot spots, which suggest a particular vitamin deficiency.

Retinol Retinol (vitamin A) is involved in epithelial cell differentiation and retinal pigmentation. Deficiency therefore results in dermatologic and ophthalmologic manifestations, including xerosis cutis and Bitot spots. It can also cause immunosuppression, which explains this patient's recurrent upper respiratory tract infections. A history of tripping at night suggests night blindness (nyctalopia), another manifestation of vitamin A deficiency.

A 38-year-old man comes to the physician because of a 2-week history of abdominal pain and an itchy rash on his buttocks. He also has fever, nausea, and diarrhea with mucoid stools. One week ago, the patient returned from Indonesia, where he went for vacation. Physical examination shows erythematous, serpiginous lesions located in the perianal region and the posterior thighs. His leukocyte count is 9,000/mm3 with 25% eosinophils. Further evaluation is most likely to show which of the following findings? This patient's fever, gastrointestinal symptoms (abdominal pain, nausea, and diarrhea), serpiginous rash, eosinophilia, and recent travel to a tropical area are highly suggestive of strongyloidiasis.

Rhabditiform larvae on stool microscopy Detection of rhabditiform larvae on stool microscopy is diagnostic of strongyloidiasis. Strongyloides stercoralis is transmitted by larval penetration of intact skin (usually when bare feet come into contact with contaminated soil). Larvae travel to the alveoli through the bloodstream, where they ascend the pulmonary system to the pharynx before being coughed up and swallowed. Once in the GI tract, they develop into adult female worms and reproduce via parthenogenesis, with the eggs hatching as rhabditiform larvae. Rhabditiform larvae then develop into infective filariform larvae, penetrating intestinal mucosa or the skin of the perianal region, which completes the process of autoinfection. This patient's serpiginous rash and gastrointestinal symptoms are manifestations of the cutaneous and intestinal phases of infection.

A 1-month-old girl is brought to the physician for evaluation of a rash on her face that first appeared 3 days ago. She was delivered at term after an uncomplicated pregnancy. She is at the 25th percentile for length and 40th percentile for weight. Examination shows small perioral vesicles surrounded by erythema and honey-colored crusts. Laboratory studies show: At birth Day 30 Hemoglobin18.0 g/dL 15.1 g/dL Leukocyte count 7,600/mm 3 6,830/mm3 Segmented neutrophils 2% 3% Eosinophils13% 10% Lymphocytes60% 63% Monocytes25% 24% Platelet count 220,000/mm 3223,000/mm3 Which of the following is the most likely diagnosis? This patient's honey-colored crusted skin rash is typical of impetigo, which can be caused by the catalase-positive bacteria Staphylococcus aureus. Patients with decreased or defective granulocyte function are susceptible to bacterial infection.

Severe congenital neutropenia Infants with severe congenital neutropenia are prone to develop recurrent bacterial infections, including otitis media, respiratory infections, or oral and skin infections secondary to staphylococcal and streptococcal pathogens (such as impetigo). Laboratory findings reveal absolute neutropenia (often as low as < 200/μL) and relative monocytosis, as seen in this patient.

A 41-year-old man is brought to the emergency department after a suicide attempt. His wife found him on the bathroom floor with an empty bottle of medication next to him. He has a history of major depressive disorder. His only medication is nortriptyline. His pulse is 127/min and blood pressure is 90/61 mm Hg. Examination shows dilated pupils and dry skin. The abdomen is distended and there is dullness on percussion in the suprapubic region. An ECG shows tachycardia and a QRS complex width of 130 ms. In addition to intravenous fluid resuscitation, which of the following is the most appropriate pharmacotherapy? This patient presents with prolonged QRS complexes (≥ 100 ms), tachycardia, hypotension, mydriasis, dry skin, and urinary retention. These symptoms are suggestive of an overdose of nortriptyline, a tricyclic antidepressant (TCA).

Sodium bicarbonate TCAs block the fast sodium channels in the myocardium, which decreases cardiac conduction velocity, resulting in prolonged QRS complexes (≥ 100 ms) and cardiac arrhythmias. In cases of TCA overdose involving arrhythmias, IV sodium bicarbonate is indicated to alkalinize plasma so that protein binding of TCAs (weak bases) prevent the drug from binding to the fast sodium channels. Additionally, sodium bicarbonate increases extracellular sodium concentration, which makes the remaining fast sodium channels more active.

A 58-year-old white man with hypertension and type 2 diabetes mellitus comes to the physician because of a 3-month history of a painless lesion on his lower lip. He has smoked one pack of cigarettes daily for 20 years. He has worked as a fruit picker for the past 25 years. His current medications include captopril and metformin. Examination of the oral cavity shows a single ulcer near the vermillion border. Which of the following is the most likely diagnosis? In patients with this condition, keratin pearls are common pathologic findings on biopsy.

Squamous cell carcinoma A chronic solitary ulcer of the lip in an older, light-skinned (white) male patient with a history of smoking and prolonged occupational sun-exposure is highly suggestive of malignancy. The most common malignancy of the lower lip is a squamous cell carcinoma (SCC), which typically manifests in chronically sun-exposed areas. 95% of malignancies of the lower lip are SCCs. SCC typically begins as a papule and then becomes a friable ulcer with everted borders that bleeds readily. Other risk factors for SCC of the lower lip include HIV and immunosuppressive therapy.

A 6-year-old boy is brought to the physician by his parents because of right lower extremity weakness, worsening headaches, abdominal pain, dark urine, and a 5-kg (11-lb) weight loss for the past 2 months. His teachers report that he has not been paying attention in class and his grades have been worsening. He has a history of infantile seizures. Physical examination shows a palpable abdominal mass and left costovertebral angle tenderness. Neurological examination shows decreased strength of the right lower limb. He has several acne-like angiofibromas around the nose and cheeks. Further evaluation is most likely to show which of the following? This boy's combination of facial angiofibromas (adenoma sebaceum), signs of angiomyolipomas (abdominal mass, macrohematuria), a history of infantile seizures, and signs of a potential brain tumor (i.e., focal neurologic deficits indicated by decreased limb strength, declining performance, poor attention) indicates tuberous sclerosis.

Subependymal giant cell astrocytoma Subependymal giant cell astrocytoma is a brain tumor associated with tuberous sclerosis (TSC). Other common manifestations of TSC that this patient could develop include shagreen patches, ash-leaf spots, benign cysts, and renal cell carcinoma.

A 29-year-old woman, gravida 1, para 0 at 11 weeks' gestation comes to the physician because of a 2-day history of left lower extremity pain and swelling. Her temperature is 37.9°C (100.2°F). Physical examination shows a tender, palpable cord on the lateral aspect of the left lower leg. The overlying skin is erythematous and indurated. Duplex ultrasound shows vascular wall thickening and subcutaneous edema. Which of the following is the most likely diagnosis? A variant of this patient's condition is Trousseau syndrome, in which palpable, tender cords recur at different locations on the body.

Superficial thrombophlebitis Superficial thrombophlebitis is usually a benign inflammation of veins and overlying tissue due to thrombosis. The inflamed veins, impacted with thrombotic material, manifest as palpable, nodular cords that remain visible when the limb is raised. There is a greater risk of thrombosis during pregnancy due to a combination of hypercoagulability (increased circulating clotting factors and RBCs) and elevated venous stasis (compression of enlarged uterus on pelvic veins).

A 16-year-old boy with a seizure disorder and cognitive delay is brought to the physician because of progressively worsening right lower extremity weakness for the past 6 months. He does not make eye contact and sits very close to his mother. Physical examination shows a grade 3/6 holosystolic murmur at the cardiac apex. Neurological examination shows decreased strength in the right lower leg with normal strength in the other extremities. Fundoscopic examination shows several multinodular, calcified lesions in the retina bilaterally. A photograph of his skin findings is shown. This patient's condition is most likely due to a mutation in which of the following? Contains overlay This patient is presenting with focal neurologic deficits, retinal hamartomas, a murmur consistent with mitral regurgitation, and ash-leaf hypopigmented lesions, all of which are characteristic of tuberous sclerosis.

TSC1 gene on chromosome 9 Tuberous sclerosis is caused by either a mutation in the TSC1 gene on chromosome 9 or a TSC2 gene mutation on chromosome 16, both of which are tumor suppressor genes. This patient presents with several common features of the condition such as cognitive development delay, focal neurological signs (due to central nervous system hamartomas or giant cell astrocytomas), and cardiac anomalies (either structural or due to cardiac rhabdomyoma). Additionally, the image displays one of the classic dermatologic manifestation, ash-leaf spots (others include angiofibromas, Shagreen spots, and ungual fibromas).

A previously healthy 29-year-old woman comes to the physician because of a 3-day history of low-grade fever, muscle aches, and a rash. She has had 4 male sexual partners over the past year and uses condoms inconsistently. Examination shows a diffuse maculopapular rash that includes the palms and soles. Testing for nonspecific anticardiolipin antibodies is positive. Which of the following is the most appropriate initial action by the physician? Testing for the presence of nonspecific anticardiolipin antibodies describes the venereal disease research laboratory (VDRL) test. In addition to the positive VDRL, this patient has a low-grade fever, myalgias, and a diffuse maculopapular rash involving the palms and soles, all of which suggest secondary syphilis.

Test for fluorescent treponemal antibody absorption Fluorescent treponemal antibody absorption test (FTA-ABS) is a confirmatory test that is performed following a positive or equivocal nontreponemal test such as the VDRL or rapid plasma reagin (RPR) test. Because treponema antibody tests usually remain positive regardless of treatment or disease activity, a positive FTA-ABS indicates either active syphilis infection or persistent antibodies from a prior infection.

An 18-month-old boy is brought to the physician by his mother because of a 2-day history of fever and a pruritic rash that started on his trunk and then progressed to his face and extremities. He has not received any childhood vaccinations because his parents believe that they are dangerous. His temperature is 38.0°C (100.4°F). A photograph of the rash is shown. Which of the following is the most likely diagnosis? A generalized pruritic rash with simultaneous occurrence of macules, papules, vesicles, and crusted lesions indicates infection with human herpesvirus 3.

Varicella Varicella (chickenpox) is caused by the varicella-zoster virus (HHV-3) and is characterized by fever and a generalized pruritic rash in various stages of healing that typically begins on the trunk and spreads to the face and extremities. Chickenpox is no longer as common in the United States following the introduction of the varicella vaccine in 1995. Children between 12-15 months of age should receive their first dose of the vaccine and a second dose between ages 4-6.


Conjuntos de estudio relacionados

Learning Module 32: Compositional Stoichiometry

View Set

2 Life Insurance build custom exam

View Set